Объем тела тройной интеграл: Вычисление тройных интегралов: теория и примеры

Содержание

Вычисление тройных интегралов: теория и примеры

  • Понятие тройного интеграла
  • Вычисление тройного интеграла путём уменьшения кратности
  • Расстановка пределов интегрирования при переходе к последовательности трёх интегралов
  • Замена переменных в тройном интеграле и цилиндрические координаты
  • Тройной интеграл в сферических координатах
  • Приложения тройного интеграла

Тройные интегралы – это аналог двойного интеграла для функции трёх переменных, заданной как f(M) = f(xyz).

Записывается тройной интеграл так:

.

Здесь V – пространственная (трёхмерная) фигура, ограниченная плоскостями, выражения которых (равенства) даны в задании вычисления тройного интеграла. V называют также замкнутой ограниченной областью трёхмерного пространства.

Вычислить тройной интеграл

— значит найти число, равное объёму тела V или, что то же самое — области V.

Практически каждый может понять смысл вычисления тройного интеграла «на своей шкуре». Точнее — «под шкурой», а ещё точнее — по своим органам дыхания — лёгким. Вне зависимости от того, знаете ли вы об этом или не знаете, в лёгких человека свыше 700 миллионов альвеол — пузырьковых образований, оплетённых сетью капилляров. Через стенки альвеол происходит газообмен. Поэтому можно рассуждать так: объём газа в лёкгих, можно представить в виде некоторой компактной области. А состоит этот объём из маленьких объёмов, сосредоточенных в альвеолах. Ключевую роль в этом сравнении играет именно огромное количество альвеол в лёгких: как мы увидим в следующем абзаце, через такое «огромное количество малостей» математически как раз и формулируется понятие тройного интеграла.

Почему именно тройной интеграл служит для нахождения объёма тела

V? Пусть область V разбита на n произвольных областей Δvi, причём под этим обозначением подразумевается не только каждая маленькая область, но и её объём. В каждой такой маленькой области выбрана произвольная точка Mi, а f(Mi) — значение функции f(M) в этой точке. Теперь будем максимально увеличивать число таких маленьких областей, а наибольший диаметр Δvi — наоборот, уменьшать. Можем составить интегральную сумму вида

.

Если функция f(M) = f(xyz) непрерывна, то будет существовать предел интегральных сумм вида, указанного выше. Этот предел и называется тройным интегралом.

В этом случае функция

f(M) = f(xyz) называется интегрируемой в области V; V — областью интегрирования; x, y, z — переменными интегрирования, dv (или dx dy dz) — элементом объёма.

Как и в случае двойных интегралов, вычисление тройных интегралов сводится к вычислению интегралов меньшей кратности.

Рассмотрим трёхмерную область V. Снизу и сверху (то есть по высоте) эта область ограничена поверхностями z = z1(xy) и z = z2(xy). С боковых сторон (то есть по ширине) область ограничена поверхностями y = y1(x) и y = y2(x). И, наконец, по глубине (если Вы смотрите на область в направлении оси Ox

) — поверхностями x = a и x = b

Чтобы применять переход к интегралам меньшей кратности, требуется, чтобы трёхмерная область V была правильной. Она правильна тогда, когда прямая, параллельная оси Oz, пересекает границу области V не более чем в двух точках. Правильными трёхмерными областями являются, например, прямоугольный параллелепипед, эллипсоид, тетраэдр. На рисунке ниже — прямоугольный параллелепипед, который встретится нам в первом примере на решение задач.

Чтобы наглядно представить отличие правильности от неправильности, добавим, что поверхности области по высоте у правильной области не должны быть вогнуты вовнутрь. На рисунке ниже — пример неправильной области V — однополостный гиперболоид, поверхность которого прямая, параллельная оси Oz (красного цвета), пересекает более чем в двух точках.

Мы будем рассматривать только правильные области.

Итак, область V — правильная. Тогда для любой функции f(xyz), непрерывной в области V, справедлива формула

Эта формула позволяет свести вычисление тройного интеграла к последовательному вычислению внутреннего определённого интеграла по переменной z (при постоянных x и y) и внешнего двойного интеграла по двумерной области D.

Переходя от двойного интеграла к повторному, получаем следующую формулу для вычисления тройного интеграла:

Таким образом, для вычисления тройного интеграла требуется последовательно вычислить три определённых интеграла.

Вычисляются эти интегралы от самого внутреннего (по переменной z) к самому внешнему (по переменной x). Для удобства восприятия последовательности вычислений три «вложенных» интеграла можно записать так:

.

Из этой записи уже однозначно видно, что:

  • сначала нужно интегрировать функцию f(xyz) по переменной z, а в качестве пределов интегрирования взять уравнения z = z1(xy) и z = z2(xy) поверхностей ограничивающих область V снизу и сверху;
  • получившийся на предыдущем шаге результат интегрировать по переменной y, а в качестве пределов интегрирования взять уравнения y = y1(x) и y = y2(x) поверхностей, ограничивающих область V с боковых сторон;
  • получившийся на предыдущем шаге результат интегрировать по переменной
    x
    , а в качестве пределов интегрирования взять уравнения x = a и x = b поверхностей, ограничивающих область V по глубине.

Пример 1. Пусть от тройного интеграла можно перейти к повторному интегралу

последовательности трёх определённых интегралов. Вычислить этот повторный интеграл.

Решение. Вычисление повторного интеграла всегда начинается с последнего интеграла. В нашем случае применяем формулу 10 из таблицы интегралов:

.

Вычислим второй интеграл — по переменной y (применяя формулу 7 из таблицы интегралов):

.

Теперь вычисляем самый внешний интеграл — по переменной

x (применяя все ту же формулу 7):

.

Ответ: данный повторный интеграл и соответствующий ему тройной интеграл равен 10.

Пример 2. Вычислить тройной интеграл

,

где V — параллелепипед, ограниченный плоскостями x = − 1, x = + 1, y = 0, y = 1, z = 0, z = 2.

Решение. Пределы интегрирования для всех трёх определённых интегралов однозначно заданы уравнениями поверхностей, ограничивающих параллелепипед. Поэтому сразу сводим данный тройной интеграл к последовательности трёх определённых интегралов:

.

Вычисляем самый внутренний интеграл — по переменной z, считая икс и игрек константами. Получаем:

.

Вычисляем интеграл «в серединке» — по переменной y. Получаем;

.

Теперь вычисляем самый внешний интеграл — по переменной x:

Ответ: данный тройной интеграл равен -2.

Пример 3. Вычислить тройной интеграл

,

где V — пирамида, ограниченная плоскостью x + y + z = 1 и координатными плоскостями x = 0, y = 0, z = 0. Область V проецируется на плоскость xOy в треугольник D, как показано на рисунке ниже.

Решение. Расставим сначала пределы интегрирования. Для интеграла по переменной z нижний предел интегрирования задан однозначно: z = 0. Чтобы получить верхний предел, выразим

z из x + y + z = 1. Получаем 1 − x − y. Для интеграла по переменной y нижний предел интегрирования задан однозначно: y = 0. Для получения верхнего предела выразим y из x + y + z = 1, считая при этом, что z = 0 (так как линия расположена в плоскости xOy). Получаем: 1 − x.

Сводим данный тройной интеграл к последовательности трёх определённых интегралов:

.

Вычисляем самый внутренний интеграл — по переменной z, считая икс и игрек константами. Получаем:

.

Вычисляем средний интеграл — по переменной y. Получаем:

Наконец, вычисляем самый внешний интеграл — по переменной x:

Ответ: данный тройной интеграл равен 1/8.

Вычислить тройной интеграл самостоятельно, а затем посмотреть решение

Пример 4. Вычислить тройной интеграл

,

где V — пирамида, ограниченная плоскостью x + y + z = 1 и координатными плоскостями x = 0, y = 0, z = 0.

Посмотреть правильное решение и ответ.

Нет времени вникать в решение? Можно заказать работу!

К началу страницы

Пройти тест по теме Интеграл

Бывает, что студенты, у которых не вызывает особых трудностей непосредственное вычисление интегралов, не могут освоиться в расстановке пределов интегрирования при переходе от тройного интеграла к последовательности трёх определённых интегралов. В этом деле действительно требуется некоторая натренированность. В первом примере область интегрирования V представляла собой параллелепипед, с которым всё понятно: со всех сторон его ограничивают плоскости, а значит, пределы интегрирования однозначно заданы уравнениями плоскостей. Во втором примере — пирамида: здесь уже требовалось чуть больше подумать и выразить один из пределов из уравнения. А если область V ограничивают не плоские поверхности? Нужно, конечно, определённым образом осмотреть область V.

Начнём с примера «пострашнее», чтобы почувствовать «обстановку, приближенную к боевой».

Пример 5. Расставить пределы интегрирования при переходе от тройного интеграла, в котором область V — эллипсоид

.

Решение. Пусть центр эллипсоида — начало координат, как показано на рисунке выше. Посмотрим на эллипсоид снизу. Снизу его ограничивает поверхность, являющаяся той части поверхности эллипсоида, которая расположена ниже плоскости xOy. Следовательно, нужно выразить из уравнения эллипсоида z и полученное выражение со знаком минус будет нижним пределом интегрирования по переменной z:

.

Теперь посмотрим на эллипсоид сверху. Здесь его ограничивает поверхность, являющаяся той части поверхности эллипсоида, которая расположена выше оси xOy. Следовательно, нужно выразить из уравнения эллипсоида z и полученное выражение будет верхним пределом интегрирования по переменной z:

.

Проекцией эллипсоида на плоскость xOy является эллипсоид. Его уравнение:

.

Чтобы получить нижний предел интегрирования по переменной y, нужно выразить y из уравнения эллипсоида и взять полученное выражение со знаком минус:

.

Для верхнего предела интегрирования по переменной y то же выражение со знаком плюс:

.

Что касается интегрирования по переменной x, то область V ограничена по глубине плоскостями. Следовательно, пределы интегрирования по переменной x можно представить как координаты задней и передней границ области. В случае эллипсоида ими будут взятые с отрицательным и положительным знаками величины длин полуоси a: x1 = − a и x2 = a.

Таким образом, последовательность интегралов для вычисления объёма эллипсоида следующая:

,

где «игрек первое», «игрек второе», «зет первое» и «зет второе» — полученные выше выражения. Если у Вас есть желание и отвага вычислить этот интеграл и, таким образом, объём эллипсоида, то вот ответ: 4πabc/3.

Следующие примеры — не такие страшные, как только что рассмотренный. При этом они предполагают не только расстановку пределов интегрирования, но и вычисление самого тройного интеграла. Проверьте, чему вы научились, следя за решением «страшного» примера. Думать при расстановке пределов всё равно придётся.

Пример 6. Вычислить тройной интеграл

,

если область интегрирования ограничена плоскостями x + y = 1, x + 2y = 4, y = 0, y = 1, z = 1, z = 5.

Решение. «Курортный» пример по сравнению с примером 5, так как пределы интегрирования по «игрек» и «зет» определены однозначно. Но придётся разобраться с пределами интегрирования по «иксу». Проекцией области интегрирования на плоскость xOy является трапеция ABCD.

В этом примере выгоднее проецировать трапецию на ось Oy, иначе, чтобы вычислить тройной интеграл, на придётся разделить фигуру на три части. В примере 4 мы начинали осмотр области интегрирования снизу, и это обычный порядок. Но в этом примере мы начинаем осмотр сбоку или, если так проще, положили фигуру набок и считаем, что смотрим на неё снизу. Можем найти пределы интегирования по «иксу» чисто алгебраически. Для этого выразим «икс» из первого и второго уравнений, данных в условии примера. Из первого уравения получаем нижний предел 1 − y, из второго — верхний 4 − 2y. Сведём данный тройной интеграл к последовательности трёх определённых интегралов:

.

Внимание! В этом примере самый внешний интеграл — не по переменной «икс», а по переменной «игрек», а «средний» — по переменной «икс»! Здесь мы применили смену порядка интегрирования, с которой ознакомились при изучении двойного интеграла. Это связано с тем, что, как уже говорилось, мы начали осмотр области интегрирования не снизу, а сбоку, то есть спроецировали её не на ось Ox, на на ось Oy.

Вычисляем самый внутренний интеграл — по переменной z, считая икс и игрек константами. Получаем:

.

Вычисляем средний интеграл — по переменной x. Получаем:

.

Наконец, вычисляем самый внешний интеграл — по переменной y:

Ответ: данный тройной интеграл равен 43.

Пример 7. Вычислить тройной интеграл

,

если область интегрирования ограничена поверхностями x = 0, y = 0, z = 2, x + y + z = 4.

Решение. Область V (пирамида MNRP) является правильной. Проекцией области V на плоскость xOy является треугольник AOB.

Нижние пределы интегрирования по всем переменным заданы в условии примера. Найдём верхний предел интегирования по «иксу». Для этого выразим «икс» из четвёртого уравнения, считая «игрек» равным нулю, а «зет» равным двум. Получаем x = 2. Найдём верхний предел интегирования по «игреку». Для этого выразим «игрек» из того же четвёртого уравнения, считая «зет» равным двум, а «икс» — переменной величиной. Получаем y = 2 − x. И, наконец, найдём верхний предел интегрирования по переменной «зет». Для этого выразим «зет» из того же четвёртого уравнения, считая «игрек» и «зет» переменными величинами. Получаем z = 4 − x − y.

Сведём данный тройной интеграл к последовательности трёх определённых интегралов:

.

Вычисляем самый внутренний интеграл — по переменной z, считая икс и игрек константами. Получаем:

.

Вычисляем средний интеграл — по переменной y. Получаем:

.

Вычисляем самый внешний интеграл — по переменной x и окончательно находим данный тройной интеграл:

Ответ: данный тройной интеграл равен 2.

Если проекцией области интегрирования на какую-либо из координатных плоскостей является круг или часть круга, то тройной интеграл проще вычислисть, перейдя к цилиндрическим координатам. Цилиндрическая система координат является обобщением полярной системы координат на пространство. В системе цилиндрических координат точка M характеризуется тремя величинами (r, φ, z), где r — расстояние от начала координат до проекции N точки M на плоскость xOy, φ — угол между вектором ON и положительным направлением оси Ox, z — аппликата точки M (рисунок ниже).

Прямоугольные координаты x, y, z с цилиндрическими координатами r, φ, z связывают формулы

x = rcosφ,

y = rsinφ,

z = z.

Для того, чтобы в тройном интеграле перейти к цилиндрическим координатам, нужно подынтегральную функцию выразить в виде функции переменных r, φ, z:

.

То есть переход от прямогольных координат к цилиндрическим осуществляется следующим образом:

.

Тройной интеграл в цилиндрических координатах вычисляется так же как и в декартовых прямоугольных координатах, путём преобразования в последовательность трёх определённых интегралов:

Пример 8. Вычислить тройной интеграл

переходом к цилиндрическим координатам, где V — область, ограниченная поверхностями и .

Решение. Так как область V на плоскость xOy проектируется в круг , то координата φ изменяется в пределах от 0 до 2π, а координата r — от r=0 до r=1. Постоянному значению в пространстве соответствует цилиндр . Рассматривая пересечение этого цилиндра с областью V, получаем изменение ординаты z от z = r² до z = 1. Переходим к цилиндрическим координатам и получаем:

Ответ: данный тройной интеграл равен π/6.

Если область интегрирования в тройном интеграле представляет собой шар или часть шара, то проще вычислить тройной интеграл в сферических координатах. В сферических координатах точку M характеризуют три величины (ρ, φ, θ), где ρ — расстояние от точки M до начала координат 0, φ — угол между вектором ON и положительным направлением оси Ox (N — проекция точки M на плоскость xOy), θ — угол между вектором OM и положительным направлением оси Oz.

Сферические координаты связаны с прямоугольными декартовыми координатами соотношениями

x = ρsinθcosφ,

y = ρsinθsinφ,

z = ρcosθ.

Элемент объёма в сферических координатах выражается следующим образом:

.

Таким образом, переход от прямоугольных декартовых координат в тройном интеграле к сферическим координатам осуществляется по формуле:

Чтобы вычислить тройной интеграл в сферических координатах, нужно поступить так же, как при вычислениях в прямоугольных декартовых и цилиндрических координатах — перейти к повторным интегралам (последовательности трёх определённых интегралов):

Пример 9. Вычислить тройной интеграл

переходом к сферическим координатам, где V — область, ограниченная неравенствами и .

Решение. Снизу область интегрирования ограничена конической поверхностью , а сверху — сферой . Так как область интегирования представляет собой часть шара, перейдём к сферическим координатам. Перепишем подынтегральную функцию:

Учитывая, что , получаем

Расставим пределы интегрирования и перепишем последний полученный интеграл в виде трёх повторных интегралов. По рисунку видно, что , , . Поэтому

Итак, тройной интеграл вычислен. Так как все три интеграла — независисмые друг от друга, мы смогли интегрировать каждый отдельно и результаты перемножить.

Вычисление объёма тела. Объём области V равен тройному интегралу по этой области, если подынтегральная функция равна 1:

.

Вычисление массы неоднородного тела. Массу неоднородного тела с плотностью ρ = ρ(xyz) можно вычислить по формуле:

.

Статические моменты материального тела. Статические моменты относительно плоскостей xOy, xOz, yOz материального тела с плотностью ρ = ρ(xyz) можно вычислить по формулам:

Моменты инерции материального тела. Моменты инерции относительно плоскостей xOy, xOz, yOz материального тела с плотностью ρ = ρ(xyz) можно вычислить по формулам:

Моменты инерции относительно осей Ox, Oy, Oz определяются по формулам:

Центр тяжести материального тела. Координаты центра массы C(xc, yc, zc) материального тела с плотностью ρ = ρ(xyz) определяются по формулам:

Пример 10. Вычислить объём тела, ограниченного поверхностями , , .

Решение. Одна их поверхностей — — цилиндрическая поверхность (образующая параллельна оси Oz), то есть проекция области на плоскость xOy совпадает с фигурой, которую ограничивает линия , или . Эта линия изображена на рисунке ниже.

Таким образом, записываем тройной интеграл в цилиндрических координатах и вычисляем его:

Ответ: объём тела равен 176 единиц объёма.

НазадЛистатьВперёд>>>

Нет времени вникать в решение? Можно заказать работу!

К началу страницы

Пройти тест по теме Кратные и криволинейные интегралы

Кратные и криволинейные интегралы

  • Вычисление двойных интегралов
  • Двойные интегралы в полярных координатах
  • Вычисление тройных интегралов
  • Вычисление криволинейных интегралов
  • Интегралы по замкнутому контуру, формула Грина
  • Вычисление поверхностных интегралов

Поделиться с друзьями

Вычисление двойных и тройных интегралов

Правильно расставить пределы интегрирования в двойном и тройном интегралах не такое тяжелое задание, особенно если выполнено построение области интегрирования или имеете представление пространственного тела. Но в большинстве случаев — на практике, контрольной или экзамене студенты не имеют возможности качественно выполнить графический анализ, визуально проанализировать области, а еще большая проблема, что многие из них не владеют техникой изменения порядка интегрирования. В отдельных задачах такой подход позволяет возвести вычисление от интегрирования по 2 -3 областям к одной. В результате быстро удается найти площадь криволинейной трапеции (фигуры на плоскости) или объема тела. Для круговых, эллиптических и разных лепестковых фигур целесообразно выполнять переход к полярным координатам, а уже в них через несколько определенных интегралов найти площадь или объем. Детальный разбор готовых ответов к указанным примерам позволяет в быстрый срок выучить методику и лучше понимать теоретический материал на кратные интегралы. Все задания взяты из индивидуальной работы для студентов ЛНУ им. І. Франко, варианты хорошо подобраны и охватывают несколько тем из высшей математики.

ВАРИАНТ — 5

ЗАДАНИЕ 1. 23 Поменять порядок интегрирования в двойном интеграле:
Решение: Методика вычислений к подобным задачам заключается в следующем:
сначала выписываем область интегрирования из интеграла, которая ограничена кривыми

Дальше превращаем функции, чтобы найти их канонический вид.
Верхний предел y=2-x2/2 — парабола с вершиной в точке O(0;2) и ветками вниз.
Возводим к каноническому виду нижний предел интегрирования

Получили полуелипс с центром в точке S (0;0) и полуосями a=2, b=3.
Изобразим кривые в декартовой системе координат (СК)
Выражаем полученные функции через переменную y:
Из первой получим корневую зависимость , перед радикалом стоит знак «+» поскольку часть параболы находится в правой (положительной по x) части плоскости;
Для эллипса будем иметь
При изменении порядка интегрирования область нужно разбить на две части: D=D1+D2.
Расставим пределы интегрирования в каждой половине:

Изменяем пределы в двойном интеграле

 

ЗАДАНИЕ 2. 24 Найти площадь плоской фигуры, заданной следующими условиями: y=x2-4x+3, y=x+1, y=-2x+5.
Решение: Задание достаточно творческое, ведь искомая фигура имеет следующий график. Но это забегая наперед
На практике Вы этого не знаете, потому начинаете анализ с поиска точек пересечения графиков заданных функций:
Сначала 1 и 2

отсюда

Дальше 2 и 3

отсюда

И напоследок 1 и 3

Координаты точек пересечения содержат корень, ето значит что преподаватель, который готовил задание особенно не старался, чтобы облегчить работу студентам и получить напоследок хороший ответ.
Заданную область можно разбивать на 2-4 части, все зависит от порядка интегрирования.
Мы же заданную область будем разбивать на две области:
D=D1+D2, как на рисунке.
Расставим пределы в каждой области:

С помощью двойного интегралу вычислим площадь фигуры, ограниченной параболой и линиями, :

Интегрировать не трудно, однако комментировать пределы во второй раз не будем.

 

ЗАДАНИЕ 3.25 Вычислить двойной интеграл по области D, ограниченной указанными линиями:
, D: x+y=1, y=x2— 1 .

Решение: Найдем точки пересечения графиков заданных функций : y=1 — x и y=x2— 1:
1 — x=x2-1, x2+x-2=0, (x-1)(x+2)=0, x=1, y=0.
Изобразим область интегрирования в декартовой СК
Расставим пределы интегрирования при переходе от параболы к прямой:

Вычислим двойной интеграл заданной функции :
.
Получили отрицательный интеграл I=-1/6.

 

ЗАДАНИЕ 4.6 Вычислить двойной интеграл, используя полярные координаты:

Решение: Сведем функции пределов интегрирования к каноническому виду

Получили круг с центром в точке O (0;0) и радиусом корень из двух (нижняя половина).
Для перехода к полярной системе координат применяем замену переменных :

При этом подинтегральную функцию следует умножить на якобиан перехода:
Превратим подинтегральную функцию под полярные координаты:

Запишем пределы интегрирования в полярной системе координат :

Вычислим двойной интеграл в полярной СК:
Он равен 0, это значит, что подинтегральная функция непарная в заданной области. Это легко видеть из ее начальной записи.

 

ЗАДАНИЕ 5.7 Вычислить площадь области D, ограниченной указанными линиями:
D: y2=4x, x2=4y.

Решение: За инструкцией находим точку пересечения двух графиков
x1=0, y1=0; x2=4; y2=4.
Расставим пределы интегрирования в заданной области D:

Построим график фигуры за известными уравнение функций
Площадь фигуры находим за формулой:
Внутренний интеграл предусматривает подстановку пределов интегрирования и только во внешнем придется использовать формулы интегрирования.
Площадь фигуры ровна S=16/3 единиц квадратных.

 

ЗАДАНИЕ 6.8 Используя двойной интеграл, вычислить, перейдя к полярным координатам, площадь плоской фигуры :

Решение: Уравнение rho=a (1 — cos (phi)) описывает кардиоиду в полярной системе координат.
График кардиоиды имеет вид
Поскольку заданная функция парная, то вычислим половину площади и результат умножим на 2.
Записываем пределы интегрирования для верхней половины фигуры :

Через двойной интеграл находим площадь кардиоиды :
Под интегралом пришлось использовать формулы понижения степени для квадрату косинуса.
Также, обратите внимание, что во всех примерах при нахождении интегралов в полярных координатах функция под интегралом должна содержать множителем якобиан перехода.

 Вычисление тройных интегралов

ЗАДАНИЕ 7.9 Найти объем тела, заданного поверхностями, что его ограничивают:
x2/9+y2/4+z2=1, x2+y2=2x, z=0

Решение: Первая поверхность x2/9+y2/4+z2=1 — эллипсоид с полуосями a=3, b=2, c=1.
Вторую сведем к каноническому виду x2+y2=2x, (x-1)2+y2=12 — коловий цилиндр вытянут вдоль оси Oz.
Запишем функции заданных поверхностей в той плоскости, где будем искать объем тела :

Построим трехмерную модель тела и его проекцию в декартовую плоскость

Объем тела, которое ограничено заданными поверхностями, найдем как разницу объемов полэллипсоида и полуцылидндра, который ограничен снизу плоскостью z=-1. При расстановке пределов интегрирования будем учитывать четность заданных функций и результат умножим на 2.
Сначала найдем объем тела, которое ограничено півеліпсоїдом.
Расставим пределы интегрирования в заданной области:

 через двойной интеграл найдем объем полэллипсоида
согласно рекомендаций, для упрощения вычислений перейдем к полярной системе координат (запишем формулы перехода и якобиан)
Найдем объем полцилиндра, который ограничен плоскостью z=-1.
Поверхность ограничена следующими пределами:

Подставляем их в формулу объема тела и рассчитываем
Конечный объем вычисляем как разницу между полуелипсом и полуцылиндром :

он равен V=3pi кубических единиц.

ЗАДАНИЕ 8.10 Расставить пределы интегрирования в тройном интеграле , если область V ограничена указанными поверхностями:

V: x=2, y=4x
Нарисовать область интегрирования.
Решение: Уравнение поверхности в пространстве запишем из последнего условия: z=y2/4 — это параболический цилиндр.
Пересечение поверхностей в пространстве и проекция тела в декартовую плоскость изображено на рисунку ниже
Пределы интегрирования расставим следующим образом:
V:
Записываем тройной интеграл с учетом найденных пределов

 

ЗАДАНИЕ 9.11 Вычислить тройные интегралы: , где V: .

Решение: Выполняем построение поверхностей интегрирования

Заданная область V является параллелепипедом, кроме этого подинтегральная функция задана в явном виде xy+2z.
Подставляем пределы в тройной интеграл и находим его значение

Формулы интегрирования для примера не тяжелые и его по силам найти всем.

 

ЗАДАНИЕ 10.12 Используя тройной интеграл, вычислить объем тела : где y=2x, y=3,
Нарисовать область интегрирования.

Решение: Сначала выполняем построение вспомогательного рисунка. Проекция тела на плоскость дает прямоугольный треугольник

Запишем пределы интегрирования, учитывая условие и выполненный рисунок :

Подставим пределы в тройной интеграл и найдем объем тела :

Само интегрирование расписано в деталях, потому проанализируйте расчеты из формулы.
На этом контрольная из высшей математики решена, выводы относительно эффективности тех или других приемов каждый должен сделать самостоятельно.
В дополнение можем лишь сказать, что Вы всегда можете обратиться к нам за консультацией относительно вычислений двойных и тройных интегралов.

Вычисление объема тела с помощью тройного интеграла

Объем тела V находится по формулам: или

–– в декартовых координатах;

–– в цилиндрических;

–– в сферических.

Пример 8.1. Найти объем тела, ограниченного поверхностями: z = x 2 + y 2 и z = 1. Сделать чертеж.

Решение. Данное тело ограничено сверху плоскостью z = 1, снизу –– параболоидом z = x 2 + y 2 (рис. 15). Объем тела находим, используя цилиндрические координаты:

(куб. ед.).

Пример 8.2. Найти объем тела, ограниченного поверхностями:

z = 4x 2 + 2y 2 + 1, x + y – 3 = 0, x = 0, y = 0, z = 0. Сделать чертеж.

Решение. Данное тело ограничено сверху частью параболоида z = 4x 2 + 2y 2 + 1, снизу –– плоскостью z = 0, боковые поверхности: x = 0, y = 0 x + y = 3 (рис. 16). Проекцией тела на плоскость xOy есть треугольник (рис. 17).

(куб. ед.).

Не нашли то, что искали? Воспользуйтесь поиском:

Лучшие изречения: Сдача сессии и защита диплома — страшная бессонница, которая потом кажется страшным сном. 8796 — | 7156 — или читать все.

78.85.5.224 © studopedia.ru Не является автором материалов, которые размещены. Но предоставляет возможность бесплатного использования. Есть нарушение авторского права? Напишите нам | Обратная связь.

Отключите adBlock!
и обновите страницу (F5)

очень нужно

В этом разделе вы найдете подробные решения, связанные и вычислением и применением тройных интегралов: от непосредственного вычисления (в декартовых, цилиндрических, сферических координатах), до применения к нахождению объемов тел, массы, моментов и т. 3=frac $$ с плотностью $z=0$ $(z ge 0)$.

Задача 11. Используя тройной интеграл в цилиндрической системе координат, вычислить массу кругового цилиндра, нижнее основание которого лежит в плоскости $xOy$, а ось симметрии совпадает с осью $Oz$, если заданы радиус основания $R$, высота цилиндра $H$ и функция плотности $gamma(
ho)$, где $
ho$ – полярный радиус точки.

Задача 12. Найти массу тела, заданного системой неравенств, если плотность тела в каждой точке задана функцией $mu$.

Задача 13. Найти момент инерции относительно оси Oz тела, ограниченного заданными поверхностями.

Примеры применения цилиндрических и сферических координат
  1. Услуги проектирования
  2. Тройной интеграл
  3. Примеры применения цилиндрических и сферических координат

Как и в случае перехода к полярным координатам в двойном интеграле, дать однозначный рецепт того, когда следует применять цилиндрические или сферические координаты, нельзя, это дело опыта. 2 > dz > ,$ используя сферические координаты.

Решение:

Область интегрирования представляет собой часть шара, расположенная в первом октанте и, следовательно, ограничена неравенствами $ < 0 le
ho le 1, >;; < 0 le varphi le frac < pi > < 2 >, > ;; < 0 le heta le frac < pi > < 2 >. > $

Далее:

Формулы. Равенство функций и эквивалентность формул. Основные эквивалентности

Вычисление криволинейного интеграла первого рода. Примеры

Определение тройного интеграла. Теорема существования тройного интеграла

Выражение площади плоской области через криволинейный интеграл

Частные случаи векторных полей

Скалярное поле, производная по направлению, градиент

Функции k-значной логики. Элементарные функции. Лемма об аналоге правила де Моргана

Вычисление криволинейного интеграла второго рода. Примеры.

Механические и физические приложения поверхностного интеграла первого рода

Вычисление площадей плоских областей

Линейный интеграл и циркуляция векторного поля

Решение задач с помощью алгебры высказываний

Вычисление криволинейного интеграла первого рода. Плоский случай

Поверхностный интеграл первого рода и его свойства

Огравление $Rightarrow $

3.3.Вычисление объемов тел с помощью тройных интегралов

Объем тела , как было отмечено в пункте 2.1., формула (2.3), равен:

,

тогда в декартовых и цилиндрических координатах, объем вычисляется по формулам:

(2.9)

(2.10)

П ример 14. Вычислить объем тетраэдра, ограниченного плоскостями:

, , и .

Решение. Построим область интегрирования, в данном случае это тетраэдр (рис. 2.20.). Проекция тетраэдра на плоскость служит треугольник, образованный прямыми

, и ,

так что изменяется от 0 до 6, а при фиксированном изменяется от 0 до (рис. 2.21.). Если же фиксированы и , и , то точка может перемещаться по вертикали от плоскости до плоскости , т.е. меняется в пределах от 0 до .

По формуле (2.6) при получаем

П ример15. Вычислить объем тела, ограниченного цилиндрической поверхностью и плоскостями , , .

Решение. Для вычисления объема тела воспользуемся формулой (2.6). Из рис. 2.22. видно, что интегрирование рационально при проецировании области в область плоскости , при этом и .

Для получения границы области исключим из уравнений параболического цилиндра и плоскости переменную и получим уравнение , которое определяет в плоскости параболу (рис. 2.23.). Таким образом, граница области состоит из дуг ветвей параболы ( и ) и отрезков прямых , . Следовательно,

Пример 16. Найти объем тела, ограниченного поверхностями: и , , .

Решение. Изобразим тело и его проекции на плоскость (рис. 2.24. и рис. 2.25.). Снизу и сверху тело ограничено плоскостями и соответственно. С боку – параболическим цилиндром и прямым круговым: , с образующими параллельными оси .

О бласть интегрирования является простой относительно оси (случай как на рис. 2.2.).

Но так как тело симметрично, то подсчитаем по формуле (2.6) только объем половины, проектирующейся в область :

.

Область будем считать простой относительно оси , по формуле (1.5) получим:

.

Пример 17. Вычислить объем тела, ограниченного поверхностями , и плоскостью .

Решение. Область , (рис. 2.26.) ограничена снизу , сверху — параболоидом , сбоку – цилиндрической поверхностью , образующие которой параллельны оси . Поэтому область проецируется на плоскость в область , ограниченную окружностью (рис. 2.27.).

Следовательно, для вычисления интеграла целесообразно перейти к цилиндрическим координатам. Применяя формулу (2.10), и учитывая, что уравнение параболоида в цилиндрической системе координат имеет вид и симметрию тела относительно обеих координатных плоск остей и получим:

.

4.Криволинейный интеграл по координатам (второго рода)

4.1.Определение и свойства криволинейного интеграла

П усть дана дуга кривой на плоскости , имеющая начало в точке А и конец в точке В (см. рис. 3.1), на которой задана пара непрерывных функций и .

Дугу АВ разобьем произвольными точками на n элементарных дуг , проекции которых на оси Ох и Оу обозначим через и . На каждой i-ой дуге выберем точку , вычислим значения функций в этой точке и и составим интегральную сумму (3.1).

(3.1)

Криволинейным интегралом по координатам (второго рода), взятым по кривой АВ, называется предел n-ой интегральной суммы (3.1) при и стремлении к нулю длины наибольшей из элементарных дуг ( при ), то есть

(3.2)

Некоторые свойства криволинейного интеграла по координатам:

1. Значение криволинейного интеграла зависит от выбора направления обхода кривой. Если изменить направление обхода, то интеграл меняет знак: .

2. Кривую интегрирования можно разбить на части точкой С, тогда .

3. Криволинейный интеграл по замкнутой кривой не зависит от выбора начальной точки на этом контуре (зависит только от направления обхода кривой).

З амечание. Криволинейный интеграл по координатам зависит от направления интегрирования. В случае, когда контур интегрирования замкнут, из двух возможных направлений обхода положительным условились называть обход его против часовой стрелки (рис. 3.2.), а интеграл 3.2. записывается в виде: .

Тройные интегралы. Вычисление объема тела

Похожие презентации:

Тройные интегралы. (Лекция 16)

Вычисление тройных интегралов

Понятие интеграла по фигуре. Выделение частных случаев: двойной интеграл, тройной интеграл. Свойства интегралов

Масса неоднородного тела. Тройной интеграл. (Лекция 2-3)

Тройной интеграл

Тройной интеграл

Вычисление тройных интегралов. Цилиндрические координаты. (Семинар 32)

Приложения тройных интегралов. (Лекция 2.4)

Тройные интегралы. Вычисление тройных интегралов. Декартовы прямоугольные координаты. (Семинар 31)

Тройные интегралы. Вычисления объёма тела

ТРОЙНЫЕ ИНТЕГРАЛЫ.
ВЫЧИСЛЕНИЕ ОБЪЕМА
ТЕЛА.
Понятие тройного интеграла вводиться
аналогично понятию двойного
интеграла.
Пусть функция f(x,y,z)  определена в
ограниченной замкнутой области T,
которая принадлежит трехмерному
пространству с определенной
декартовой системой координат Oxyz.
 Разобьем заданную область на n
частей, которые не имеют общих
внутренних точек и объемы которых
равны соответственно.
В каждой такой элементарной области
возьмем произвольную точку Pi(xi,yi,zi)
n
составим интегральную
сумму ∑f(xi,yi,zi)dVi
Тройной интеграл в общем виде записывается следующим
образом:
f(x,y,z) – подынтегральная функция трех переменных.
dxdydz – произведение дифференциалов.
T – область интегрирования – пространственное тело
ограниченное множеством поверхностей.
Вычислить тройной интеграл – это значит найти ЧИСЛО:
В соответствии с общим смыслом интегрирования,
произведение dxdydz равно бесконечно малому объему dV
элементарного тела.
Тройной интеграл объединяет все эти бесконечно малые
частички по области , в результате чего получается
интегральное (суммарное) значение объёма тела:
Как решать тройной
интеграл?
Пример 1.
С помощью тройного интеграла вычислить объем тела,
ограниченного поверхностями
Варианты ответа:
1)
2)
3)
4)
С помощью тройного интеграла вычислить объем тела,
ограниченного поверхностями
1)используем формулу 
Сначала
изобразим параллельную ортогональную проекцию тела на
координатную плоскость XOY.
2) выясняем, чем тело ограничено с сверху, чем снизу и
выполняем пространственный чертёж.
z=y² параболический цилиндр
расположенный
над плоскостью XOY и
проходящий через
ось OX:
3)Выбираем порядок обхода тела: Двигаемся по OZ
Двигаемся по OY
=>
Двигаемся по OX
Решение свелось к двойному интегралу, используем
формулу:
Ответ: 1)
Пример 2.
Вычислить с помощью тройного интеграла объем тела,
ограниченного указанными поверхностями. Выполнить
чертёж.
Варианты ответа:
1)
2)
3)
4)
Решим систему
 получены
две прямые, лежащие в плоскости
параллельные оси
Изобразим проекцию тела на плоскость
XOY:
Искомое тело ограниченно
плоскостью z=0
снизу и
параболическим цилиндром
z=1-x² сверху:
Составим порядок обхода тела: Двигаемся по OZ
Двигаемся по OY
Двигаемся по OX
При интегрировании по «игрек» – «икс» считается константой,
поэтому константу целесообразно сразу вынести за знак интеграла.
Ответ: 2)
Пример 3.
Вычислить с помощью тройного интеграла объём тела,
ограниченного поверхностями 
Выполнить чертёжи данного тела и его проекции на
плоскость XOY.
Варианты отета:
1)
2)
3)
4)
Решение: придерживаемся того же порядка действий: в
первую очередь рассматриваем уравнения, в которых
отсутствует переменная «зет». Оно здесь одно.
Проекция цилиндрической поверхности
на
плоскость
представляет собой «одноимённую» окружность.
Плоскости
 ограничивают искомое тело снизу и
сверху («высекают» его из цилиндра) и проецируются в круг
Плоскость
пересекает цилиндр
под косым
углом, в результате чего получается эллипс.
Из уравнения
вычислим значения функции
(«высоту») в напрашивающихся точках
Проекция тела на плоскость XOY представляет собой круг, и это
весомый аргумент в пользу перехода к цилиндрической системе
координат:
Найдём уравнения поверхностей в цилиндрических координатах:
порядок обхода тела:
Ответ: 3)
Пример 4.
С помощью тройного интеграла вычислить объём заданного
тела:
, где   – произвольное положительное
число.
 неравенство  
задаёт шар с центром в начале
координат радиуса  , а неравенство

«внутренность» кругового цилиндра с осью симметрии
радиуса
.
Таким образом, искомое тело ограничено
круговым цилиндром сбоку и симметричными относительно
плоскости  
сферическими сегментами сверху и снизу.  
Варианты ответа:
1)
2)
3)
4)
Порядок обхода:
Решаем методом подведения под знак дифференциала:
Ответ: 4)

English     Русский Правила

Тройные интегралы. Вычисление объема тела презентация, доклад

Слайд 1
Текст слайда:

ТРОЙНЫЕ ИНТЕГРАЛЫ. ВЫЧИСЛЕНИЕ ОБЪЕМА ТЕЛА.


Слайд 2
Текст слайда:

Понятие тройного интеграла вводиться аналогично понятию двойного интеграла.
Пусть функция f(x,y,z)  определена в ограниченной замкнутой области T, которая принадлежит трехмерному пространству с определенной декартовой системой координат Oxyz.  Разобьем заданную область на n частей, которые не имеют общих внутренних точек и объемы которых равны соответственно.
В каждой такой элементарной области возьмем произвольную точку Pi(xi,yi,zi)
n
составим интегральную сумму ∑f(xi,yi,zi)dVi
i=1


Слайд 3
Текст слайда:

Тройной интеграл в общем виде записывается следующим образом:

f(x,y,z) – подынтегральная функция трех переменных.
dxdydz – произведение дифференциалов.
T – область интегрирования – пространственное тело ограниченное множеством поверхностей.
Вычислить тройной интеграл – это значит найти ЧИСЛО:

В соответствии с общим смыслом интегрирования, произведение dxdydz равно бесконечно малому объему dV элементарного тела.
Тройной интеграл объединяет все эти бесконечно малые частички по области , в результате чего получается интегральное (суммарное) значение объёма тела:


Слайд 4
Текст слайда:

Как решать тройной интеграл?

Пример 1.
С помощью тройного интеграла вычислить объем тела, ограниченного поверхностями Варианты ответа:
1) 2) 3) 4)
С помощью тройного интеграла вычислить объем тела, ограниченного поверхностями
1)используем формулу  Сначала изобразим параллельную ортогональную проекцию тела на координатную плоскость XOY.
2) выясняем, чем тело ограничено с сверху, чем снизу и выполняем пространственный чертёж.
z=y² параболический цилиндр расположенный
над плоскостью XOY и проходящий через
ось OX:


Слайд 5
Текст слайда:

3)Выбираем порядок обхода тела: Двигаемся по OZ
Двигаемся по OY =>
Двигаемся по OX

Решение свелось к двойному интегралу, используем формулу:

Ответ: 1)


Слайд 6
Текст слайда:

Пример 2.
Вычислить с помощью тройного интеграла объем тела, ограниченного указанными поверхностями. Выполнить чертёж.
Варианты ответа:
1) 2) 3) 4)
Решим систему  получены

две прямые, лежащие в плоскости параллельные оси
Изобразим проекцию тела на плоскость XOY:
Искомое тело ограниченно плоскостью z=0
снизу и
параболическим цилиндром z=1-x² сверху:


Слайд 7
Текст слайда:

Составим порядок обхода тела: Двигаемся по OZ
Двигаемся по OY
Двигаемся по OX

При интегрировании по «игрек» – «икс» считается константой, поэтому константу целесообразно сразу вынести за знак интеграла.

Ответ: 2)


Слайд 8
Текст слайда:

Пример 3.
Вычислить с помощью тройного интеграла объём тела, ограниченного поверхностями 
Выполнить чертёжи данного тела и его проекции на плоскость XOY.
Варианты отета:
1) 2) 3) 4)
Решение: придерживаемся того же порядка действий: в первую очередь рассматриваем уравнения, в которых отсутствует переменная «зет». Оно здесь одно. Проекция цилиндрической поверхности на плоскость
представляет собой «одноимённую» окружность.
Плоскости  ограничивают искомое тело снизу и сверху («высекают» его из цилиндра) и проецируются в круг
Плоскость пересекает цилиндр под косым углом, в результате чего получается эллипс.
Из уравнения вычислим значения функции («высоту») в напрашивающихся точках


Слайд 9
Текст слайда:

Проекция тела на плоскость XOY представляет собой круг, и это весомый аргумент в пользу перехода к цилиндрической системе координат:

Найдём уравнения поверхностей в цилиндрических координатах:

порядок обхода тела:

Ответ: 3)


Слайд 10
Текст слайда:

Пример 4.
С помощью тройного интеграла вычислить объём заданного тела: , где   – произвольное положительное число.
 неравенство   задаёт шар с центром в начале координат радиуса  , а неравенство – «внутренность» кругового цилиндра с осью симметрии радиуса . Таким образом, искомое тело ограничено круговым цилиндром сбоку и симметричными относительно плоскости   сферическими сегментами сверху и снизу. 
Варианты ответа:
1) 2) 3) 4)

Порядок обхода:


Слайд 11
Текст слайда:

Решаем методом подведения под знак дифференциала:

Ответ: 4)


Скачать презентацию

Расчет объемов с использованием тройных интегралов

Объем твердого тела U в декартовых координатах xyz равен

\[V = \iiint\limits_U {dxdydz} . 2}\sin \theta d\rho d\varphi d\theta} .\] 92}.\]

Пример 3

Найти объем тетраэдра, ограниченного плоскостями, проходящими через точки \(A\left( {1,0,0} \right),\) \(B\left( {0,2,0} \right),\) \(C\left( {0,0,3} \right)\) и координатные плоскости \(Oxy,\) \(Oxz,\) \(Oyz \) \(\left({\text{Рисунок }2}\right).\)

Пример 4

Найти объем тетраэдра, ограниченного плоскостями

\[x + y + z = 5, x = 0, y = 0, z = 0\]

(рис. \(4\)).

Пример 1. 93}}}{3}.\]

В результате получаем известное выражение для объема шара радиуса \(R.\)

Пример 3.

Найти объем тетраэдра, ограниченного плоскостями, проходящими через точки \(A\left( {1,0,0} \right),\) \(B\left( {0,2, 0} \right),\) \(C\left( {0,0,3} \right)\) и координатных плоскостях \(Oxy,\) \(Oxz,\) \(Oyz\) \(\ слева ({\ текст {Рисунок} 2} \ справа). \)

Раствор.

Уравнение прямой \(AB\) в \(xy\)-плоскости (рис. \(3\)) записывается в виде \(y = 2 — 2x. \) Переменная \(x\) принимает значения здесь в интервале \(0 \le x \le 1,\), а переменная \(y\) находится в интервале \(0 \le y \le 2 — 2x.\)

Рис. 2. Рис. 3.

Запишем уравнение плоскости \(ABC\) в виде отрезка. Поскольку плоскость \(ABC\) пересекает отрезки \(1, 2,\) и \(3,\) соответственно, на \(x-,\) \(y-,\) и \(z- \)оси, то ее уравнение можно записать в виде

\[\frac{x}{1} + \frac{y}{2} + \frac{z}{3} = 1.\]

В общем случае уравнение плоскости \(ABC\) записывается как

\[6x + 3y + 2z = 6\;\;\text{or}\;\; z = 3 — 3x — \frac{3}{2}y.\]

Следовательно, пределы интегрирования по переменной \(z\) лежат в интервале от \(z = 0\) до \(z = 3 — 3x — {\frac{3}{2}} y.\) 93}}}{3}} \справа) = 1.\]

Пример 4.

Найти объем тетраэдра, ограниченного плоскостями

\[x + y + z = 5, x = 0, y = 0, z = 0\]

(рис. \(4\) ).

Раствор.

Уравнение плоскости \(x + y + z = 5\) можно переписать в виде

\[г = 5 — х — у. \]

Установив \(z = 0,\), мы получим

\[5 — х — у = 0\;\;\текст{или}\;\;у = 5 — х.\]

Следовательно, область интегрирования \(D\) в плоскости \(xy\) ограничена прямой линией \(y = 5 — x\), как показано на рис. \(5.\) 95} \справа] = \frac{1}{2}\left( {125 — 5 \cdot 25 + \frac{{125}}{3}} \right) = \frac{{125}}{6}.\]

Дополнительные проблемы см. на стр. 2.

HartleyMath — Тройные интегралы

Тройные интегралы труднее (фактически, обычно невозможно) изобразить, чем двойные интегралы, но принцип во многом тот же. Вместо интегралов типа ∬Rf(x,y) dA,\iint_R f(x,y) \ dA,∬R​f(x,y) dA, теперь мы будем обрабатывать интегралы, такие как ∭Df(x,y,z) dV\iiint_D f(x,y,z)\ dV∭D​f(x,y,z) dV где DDD — 333-мерное тело. 92 6\ dx = \ans{12} \end{aligned}∫02​∫03​∫02​ dx dy dx​=∫02​∫03​2 dy dx=∫02​6 dx=12​​

Обратите внимание, что это дает объем 2× Куб 3×22×3×22×3×2.

Используйте тройной интеграл, чтобы найти объем призмы DDD в первом октанте, ограниченной плоскостями y=4−2xy=4−2xy=4−2x и z=6z=6z=6. {12 -2x-3y} \ dz\ dy\ dx = 8∫04∫06−3y/2∫012−2x−3y dz dx dy=∫06∫04−2x/3∫012−2x−3y​ dz dy dx=8 91 = \ans{9\ кг} \end{align}M​=∫01​∫02​∫03​2−z dx dy dz=∫01​∫02​6−3z dy dz=∫01​12−6z dz=12z−3z2∣∣∣ ∣​01​=9 кг​​

Среднее значение

Среднее значение f(x,y,z)f(x,y,z)f(x,y,z) составляет f‾=1объем D∭Df(x,y,z) dV\overline{f} = \dfrac{1}{\textrm{объем } D} \iiint_D f(x,y,z)\ dVf​= объем D1​∭D​f(x,y,z) dV

Найдите среднее значение f(x,y,z)=250xysin⁡(πz)f(x,y,z)=250xy \sin (πz)f(x,y,z)=250xysin(πz) над кубом D={(x,y,z) : 0≤x≤2, 0≤y≤2, 0≤z≤1}.D={(x,y,z)\ :\ 0 \leq x \leq 2 ,\ 0 \leq y \leq 2,\ 0 \leq z \leq 1}.D={(x,y,z) : 0≤x≤2, 0≤y≤2, 0≤z≤1}. 92 = \ans{4} \end{выровнено}∫02​∫04−y2​∫0y​​2x dz dx dy=∫02​∫04-y2​​2xy dx dy=∫02​(4−y2)y dy=2y-41 ​y4∣∣∣∣​02​=4​​

Найдите объем тела в первом октанте, образованном, когда цилиндр z=sin⁡yz=\sin yz=siny при 0≤y≤π0≤y≤ π0≤y≤π разбивается плоскостями y=xy=xy=x и x=0x=0x=0.

Решение

Обратите внимание, что D={(x,y,z) : 0≤x≤y, 0≤y≤π, 0≤z≤sin⁡y}. D={(x,y,z)\ :\ 0 \leq x \leq y,\ 0 \leq y \leq \pi,\ 0 \leq z \leq \sin y}.D={(x,y,z) : 0≤x≤y, 0 ≤y≤π, 0≤z≤siny}.

9\pi \ \ \ \textrm{ (с использованием интеграции по частям)}\ &= \ans{\pi} \end{align}V​=∫0π​∫0y​∫0siny​ dz dx dy=∫0π​∫0y​siny dx dy=∫0π​ysiny dy=−ycosy+siny∣∣∣∣​0π​    (используя Интегрирование по частям)=π​​

Найдите объем призмы в первом октанте, ограниченный координатами z=2−4xz=2−4xz=2−4x и y=8y=8y=8.

Решение

Обратите внимание, что D={(x,y,z) : 0≤x≤1/2, 0≤y≤8, 0≤z≤2−4x}.D={(x,y,z)\ :\ 0 \leq x \leq 1/2,\ 0 \leq y \leq 8,\ 0 \leq z \leq 2-4x}.D={(x,y,z) : 0≤x≤1/2, 0≤y ≤8, 0≤z≤2−4x}. 9{\ sqrt {3}} = \ ans {\ dfrac {24 \ sqrt {3}} {5}} \end{align}V​=∫−3​3​∫x23​∫03–y​ dz dy dx=∫−3​3​∫x23​3-y dy dx=∫−3​3​​ 29​−3×2+21​x4 dx=29​x−x3+101​x5∣∣∣∣​−3​3​=5243​​​​

  1. Вычислите интеграл ∭Dxy+xz+yz dV \displaystyle\iiint_D xy+xz+yz\ dV∭D​xy+xz+yz dV, где D={(x,y,z) : −1≤x≤1,−2≤y≤2,−1≤ z≤3}D = {(x,y,z)\ :\ -1 \leq x \leq 1, -2 \leq y \leq 2, -1 \leq z \leq 3}D={(x, y,z): −1≤x≤1,−2≤y≤2,−1≤z≤3}. 2 =1.\кр }$$ $\квадрат$ 92) \; dV$ является максимальным.

    5.5 Тройные интегралы в цилиндрических и сферических координатах – исчисление, том 3

    Цели обучения

    • 5.5.1 Вычислите тройной интеграл, перейдя к цилиндрическим координатам.
    • 5.5.2 Вычислите тройной интеграл, перейдя к сферическим координатам.

    Ранее в этой главе мы показали, как преобразовать двойной интеграл в прямоугольных координатах в двойной интеграл в полярных координатах для более удобного решения задач, связанных с круговой симметрией. Аналогичная ситуация возникает и с тройными интегралами, но здесь нужно различать цилиндрическую симметрию и сферическую симметрию. В этом разделе мы преобразуем тройные интегралы в прямоугольных координатах в тройные интегралы либо в цилиндрических, либо в сферических координатах.

    Также вспомните вступительную главу, в которой был показан оперный театр l’Hemisphèric в Валенсии, Испания. Он состоит из четырех секций, одна из которых представляет собой театр в пятиэтажной сфере (мяче) под овальной крышей длиной с футбольное поле. Внутри находится экран IMAX, который превращает сферу в планетарий с небом, полным

    000 мерцающих звезд. Используя тройные интегралы в сферических координатах, мы можем найти объемы различных геометрических фигур, подобных этим.

    Обзор цилиндрических координат

    Как мы видели ранее, в двумерном пространстве ℝ2,ℝ2 точка с прямоугольными координатами (x,y)(x,y) может быть отождествлена ​​с (r,θ)(r,θ) в полярных координатах и ​​наоборот наоборот, где x=rcosθ,x=rcosθ,y=rsinθ,y=rsinθ,r2=x2+y2r2=x2+y2 и tanθ=(yx)tanθ=(yx) — отношения между переменными.

    В трехмерном пространстве ℝ3,ℝ3 точку с прямоугольными координатами (x,y,z)(x,y,z) можно отождествить с цилиндрическими координатами (r,θ,z)(r,θ,z) и наоборот. Мы можем использовать те же отношения преобразования, добавив zz в качестве вертикального расстояния до точки от плоскости xyxy, как показано на следующем рисунке.

    Рисунок 5,50 Цилиндрические координаты аналогичны полярным координатам с добавленной вертикальной координатой zz.

    Чтобы преобразовать прямоугольные координаты в цилиндрические, мы используем преобразование x=rcosθx=rcosθ и y=rsinθ.y=rsinθ. Чтобы преобразовать цилиндрические координаты в прямоугольные, мы используем r2=x2+y2r2=x2+y2 и θ=tan−1(yx).θ=tan−1(yx). Координата zz остается одинаковой в обоих случаях.

    В двумерной плоскости с прямоугольной системой координат, когда мы говорим x=kx=k (постоянная), мы имеем в виду неограниченную вертикальную линию, параллельную оси yy, а когда y=ly=l (постоянная) мы имеем в виду неограниченную горизонтальная линия, параллельная оси xx. В полярной системе координат, когда мы говорим r=cr=c (постоянная), мы имеем в виду окружность радиуса cc единиц, а когда θ=αθ=α (постоянная) мы имеем в виду бесконечный луч, образующий угол αα с положительной xx- ось.

    Аналогично, в трехмерном пространстве с прямоугольными координатами (x,y,z),(x,y,z) уравнения x=k,y=l,x=k,y=l и z=m ,z=m, где k,l,k,l, и mm являются константами, представляют собой неограниченные плоскости, параллельные плоскости yzyz, плоскости xzxz и плоскости xyxy соответственно. С цилиндрическими координатами (r,θ,z),(r,θ,z), по r=c,θ=α,r=c,θ=α и z=m,z=m, где c,α,c,α , и mm — константы, мы имеем в виду неограниченный вертикальный цилиндр с осью zz в качестве его радиальной оси; плоскость, составляющая постоянный угол αα с плоскостью xyxy; и неограниченная горизонтальная плоскость, параллельная плоскости xyxy, соответственно. Это означает, что круговой цилиндр x2+y2=c2x2+y2=c2 в прямоугольных координатах может быть представлен просто как r=cr=c в цилиндрических координатах. (Подробнее см. в разделе Цилиндрические и сферические координаты.)

    Интеграция в цилиндрических координатах

    Тройные интегралы часто проще вычислить, используя цилиндрические координаты вместо прямоугольных. Некоторые общие уравнения поверхностей в прямоугольных координатах вместе с соответствующими уравнениями в цилиндрических координатах перечислены в таблице 5.1. Эти уравнения пригодятся, когда мы приступим к решению задач с использованием тройных интегралов.

    Круглый цилиндр Круглый конус Сфера Параболоид
    Прямоугольный х2+у2=с2х2+у2=с2 z2=c2(x2+y2)z2=c2(x2+y2) х2+у2+z2=с2х2+у2+z2=с2 г=с(х2+у2)г=с(х2+у2)
    Цилиндрический г=кр=с г=крз=кр r2+z2=c2r2+z2=c2 г=cr2z=cr2

    Стол 5. 1 Уравнения некоторых распространенных фигур

    Как и прежде, начнем с простейшей ограниченной области BB в ℝ3,ℝ3, чтобы описать в цилиндрических координатах в виде цилиндрического ящика, B={(r,θ,z)|a≤r≤b,α≤ θ≤β,c≤z≤d}B={(r,θ,z)|a≤r≤b,α≤θ≤β,c≤z≤d} (рис. 5.51). Предположим, что мы делим каждый интервал на l,mandnl,mandn подразделений так, что . Тогда мы можем сформулировать следующее определение тройного интеграла в цилиндрических координатах.

    Рисунок 5,51 Цилиндрический ящик BB, описываемый цилиндрическими координатами.

    Определение

    Рассмотрим цилиндрический ящик (выраженный в цилиндрических координатах)

    B={(r,θ,z)|a≤r≤b,α≤θ≤β,c≤z≤d}.B={(r, θ,z)|a≤r≤b,α≤θ≤β,c≤z≤d}.

    Если функция f(r,θ,z)f(r,θ,z) непрерывна на BB и если (rijk*,θijk*,zijk*)(rijk*,θijk*,zijk*) является любой выборкой точка в цилиндрическом подбоксе Bijk=[ri−1,ri]×[θj−1,θj]×[zk−1,zk]Bijk=[ri−1,ri]×[θj−1,θj]×[zk −1,zk] (рис. 5.51), то мы можем определить тройной интеграл в цилиндрических координатах как предел тройной суммы Римана, если существует следующий предел:

    liml,m,n→∞∑i=1l∑j=1m∑k=1nf(rijk*,θijk*,zijk*)rijk*ΔrΔθΔz. liml,m,n→∞∑i=1l∑j=1m ∑k=1nf(rijk*,θijk*,zijk*)rijk*ΔrΔθΔz.

    Заметим, что если g(x,y,z)g(x,y,z) функция в прямоугольных координатах, а прямоугольник BB выражен в прямоугольных координатах, то тройной интеграл ∭Bg(x,y,z)dV ∭Bg(x,y,z)dV равен тройному интегралу ∭Bg(rcosθ,rsinθ,z)rdrdθdz∭Bg(rcosθ,rsinθ,z)rdrdθdz, и мы имеем

    ∭Bg(x,y,z)dV=∭Bg(rcosθ,rsinθ,z)rdrdθdz=∭Bf(r,θ,z)rdrdθdz.∭Bg(x,y,z)dV=∭Bg(rcosθ, rsinθ,z)rdrdθdz=∭Bf(r,θ,z)rdrdθdz.

    (5.11)

    Как упоминалось в предыдущем разделе, все свойства двойного интеграла хорошо проявляются в тройных интегралах, как в прямоугольных, так и в цилиндрических координатах. Они справедливы и для повторных интегралов. Повторим, что в цилиндрических координатах теорема Фубини принимает следующий вид:

    Теорема 5.12

    Теорема Фубини в цилиндрических координатах

    Предположим, что g(x,y,z)g(x,y,z) непрерывна на прямоугольном блоке B,B, который при описании в цилиндрических координатах выглядит как B={(r,θ,z)|a ≤r≤b,α≤θ≤β,c≤z≤d}. B={(r,θ,z)|a≤r≤b,α≤θ≤β,c≤z≤d}.

    Тогда g(x,y,z)=g(rcosθ,rsinθ,z)=f(r,θ,z)g(x,y,z)=g(rcosθ,rsinθ,z)=f(r ,θ,z) и

    ∭Bg(x,y,z)dV=∫cd∫αβ∫abf(r,θ,z)rdrdθdz.∭Bg(x,y,z)dV=∫cd∫αβ∫ abf(r,θ,z)rdrdθdz.

    Повторный интеграл может быть эквивалентно заменен любым из пяти других повторных интегралов, полученных путем интегрирования по трем переменным в других порядках.

    Цилиндрические системы координат хорошо подходят для тел, симметричных относительно оси, таких как цилиндры и конусы. Давайте рассмотрим несколько примеров, прежде чем мы определим тройной интеграл в цилиндрических координатах на общих цилиндрических областях.

    Пример 5,43

    Вычисление тройного интеграла по цилиндрическому ящику

    Вычисление тройного интеграла ∭B(zrsinθ)rdrdθdz∭B(zrsinθ)rdrdθdz, где цилиндрический ящик BB равен B={(r,θ,z)|0≤r≤2 ,0≤θ≤π/2,0≤z≤4}.B={(r,θ,z)|0≤r≤2,0≤θ≤π/2,0≤z≤4}.

    Решение

    Как указано в теореме Фубини, мы можем записать тройной интеграл в виде повторного интеграла . ∭B(zrsinθ)rdrdθdz=∫θ=0θ=π/2∫r=0r=2∫z=0z=4(zrsinθ)rdzdrdθ.

    Вычисление повторного интеграла выполняется просто. Каждая переменная в интеграле независима от других, поэтому мы можем интегрировать каждую переменную отдельно и перемножать результаты вместе. Это значительно упрощает вычисления:

    ∫θ=0θ=π/2∫r=0r=2∫z=0z=4(zrsinθ)rdzdrdθ=(∫0π/2sinθdθ)(∫02r2dr)(∫04zdz)=( −cosθ|0π/2)(r33|02)(z22|04)=643.∫θ=0θ=π/2∫r=0r=2∫z=0z=4(zrsinθ)rdzdrdθ=(∫0π/2sinθdθ )(∫02r2dr)(∫04zdz)=(−cosθ|0π/2)(r33|02)(z22|04)=643.

    Контрольно-пропускной пункт 5.27

    Вычислить тройной интеграл рдздрдθ.

    Если цилиндрическая область, по которой мы должны интегрировать, является общим телом, мы смотрим на проекции на координатные плоскости. Следовательно, тройной интеграл непрерывной функции f(r,θ,z)f(r,θ,z) по сплошной области общего вида E={(r,θ,z)|(r,θ)∈D,u1( r,θ)≤z≤u2(r,θ)}E={(r,θ,z)|(r,θ)∈D,u1(r,θ)≤z≤u2(r,θ)} в ℝ3,ℝ3, где DD — проекция EE на плоскость rθrθ, равно

    ∭Ef(r,θ,z)rdrdθdz=∬D[∫u1(r,θ)u2(r,θ)f(r,θ,z)dz]rdrdθ. ∭Ef(r,θ,z)rdrdθdz =∬D[∫u1(r,θ)u2(r,θ)f(r,θ,z)dz]rdrdθ.

    В частности, если D={(r,θ)|g1(θ)≤r≤g2(θ),α≤θ≤β},D={(r,θ)|g1(θ)≤r≤g2 (θ),α≤θ≤β}, то имеем

    ∭Ef(r,θ,z)rdrdθ=∫θ=αθ=β∫r=g1(θ)r=g2(θ)∫z=u1(r,θ)z=u2(r,θ)f( r,θ,z)rdzdrdθ.∭Ef(r,θ,z)rdrdθ=∫θ=αθ=β∫r=g1(θ)r=g2(θ)∫z=u1(r,θ)z=u2 (r,θ)f(r,θ,z)rdzdrdθ.

    Аналогичные формулы существуют и для проекций на другие координатные плоскости. При необходимости мы можем использовать полярные координаты в этих плоскостях.

    Пример 5,44

    Построение тройного интеграла в цилиндрических координатах по общей области

    Рассмотрим область EE внутри правого кругового цилиндра с уравнением r=2sinθ,r=2sinθ, ограниченную снизу плоскостью rθrθ и ограниченную сверху сферой радиуса 44 с центром в начале координат (рис. 5.52). Задайте тройной интеграл по этой области с функцией f(r,θ,z)f(r,θ,z) в цилиндрических координатах.

    Рисунок 5,52 Составление тройного интеграла в цилиндрических координатах по цилиндрической области.

    Решение

    Во-первых, определите, что уравнение сферы имеет вид r2+z2=16.r2+z2=16. Мы видим, что пределы для zz лежат в пределах от 00 до z=16−r2.z=16−r2. Тогда пределы для rr составляют от 00 до r=2sinθ.r=2sinθ. Наконец, пределы для θθ лежат в пределах от 00 до π,π. Следовательно, область равна

    E={(r,θ,z)|0≤θ≤π,0≤r≤2sinθ,0≤z≤16−r2}.E={(r,θ,z)|0 ≤θ≤π,0≤r≤2sinθ,0≤z≤16−r2}.

    Следовательно, тройной интеграл равен

    ∭Ef(r,θ,z)rdzdrdθ=∫θ=0θ=π∫r=0r=2sinθ∫z=0z=16−r2f(r,θ,z)rdzdrdθ. ∭Ef(r,θ,z)rdzdrdθ=∫θ=0θ=π∫r=0r=2sinθ∫z=0z=16−r2f(r,θ,z)rdzdrdθ.

    Контрольно-пропускной пункт 5,28

    Рассмотрим область EE внутри правого кругового цилиндра с уравнением r=2sinθ,r=2sinθ, ограниченную снизу плоскостью rθrθ и ограниченную сверху z=4−y.z=4−y. Задайте тройной интеграл с функцией f(r,θ,z)f(r,θ,z) в цилиндрических координатах.

    Пример 5,45

    Составление тройного интеграла в двух направлениях

    Пусть EE будет областью, ограниченной снизу конусом z=x2+y2z=x2+y2 и сверху параболоидом z=2−x2−y2. z=2−x2− у2. (рис. 5.53). Настройте тройной интеграл в цилиндрических координатах, чтобы найти объем области, используя следующие порядки интегрирования:

    1. дздрдθдздрдθ
    2. drdzdθ.drdzdθ.

      Рисунок 5,53 Составление тройного интеграла в цилиндрических координатах по конической области.

    Решение
    1. Конус имеет радиус 1 в месте пересечения с параболоидом. Поскольку z=2−x2−y2=2−r2z=2−x2−y2=2−r2 и z=x2+y2=rz=x2+y2=r (при условии, что rr неотрицательно), мы имеем 2−r2=r .2−r2=r. Решая, имеем r2+r−2=(r+2)(r−1)=0.r2+r−2=(r+2)(r−1)=0. Так как r≥0,r≥0, то r=1.r=1. Поэтому z=1.z=1. Таким образом, пересечение этих двух поверхностей представляет собой окружность радиуса 11 в плоскости z=1.z=1. Конус — это нижняя граница для zz, а параболоид — верхняя граница. Проекция области на плоскость xyxy представляет собой окружность радиусом 11 с центром в начале координат.
      Таким образом, мы можем описать область как

      E={(r,θ,z)|0≤θ≤2π,0≤r≤1,r≤z≤2−r2}. E={(r,θ, z)|0≤θ≤2π,0≤r≤1,r≤z≤2−r2}.


      Следовательно, интеграл для объема равен

      rz=2−r2rdzdrdθ.

    2. Мы также можем записать поверхность конуса как r=zr=z, а параболоид как r2=2−z.r2=2−z. Нижняя граница для rr равна нулю, но верхняя граница иногда представляет собой конус, а иногда — параболоид. Плоскость z=1z=1 делит область на две области. Тогда область может быть описана как

      E={(r,θ,z)|0≤θ≤2π,0≤z≤1,0≤r≤z}∪{(r,θ,z)|0≤θ≤2π,1≤z≤ 2,0≤r≤2−z}.E={(r,θ,z)|0≤θ≤2π,0≤z≤1,0≤r≤z}∪{(r,θ,z)| 0≤θ≤2π,1≤z≤2,0≤r≤2−z}.


      Теперь интеграл для объема принимает вид

      zrdrdzdθ.V=∫θ=0θ=2π∫z=0z=1∫r=0r=zrdrdzdθ+∫θ=0θ=2π∫z=1z=2∫r=0r=2−zrdrdzdθ.

    Контрольно-пропускной пункт 5.29

    Повторить предыдущий пример с порядком интегрирования dθdzdr.dθdzdr.

    Пример 5,46

    Нахождение объема с помощью тройных интегралов двумя способами

    Пусть E — область, ограниченная снизу плоскостью rθrθ, сверху — сферой x2+y2+z2=4,×2+y2+z2=4, а на сторонами цилиндра x2+y2=1×2+y2=1 (рис. 5.54). Задайте тройной интеграл в цилиндрических координатах, чтобы найти объем области, используя следующие порядки интегрирования, и в каждом случае найдите объем и убедитесь, что ответы совпадают:

    1. dzdrdθdzdrdθ
    2. drdzdθ.drdzdθ.

      Рисунок 5,54 Нахождение цилиндрического объема тройным интегралом в цилиндрических координатах.

    Решение
    1. Обратите внимание, что уравнение для сферы

      x2+y2+z2=4orr2+z2=4×2+y2+z2=4orr2+z2=4


      , а уравнение для цилиндра

      x2+y2=1orr2=1. х2+у2=1или2=1.


      Таким образом, для области EE

      E={(r,θ,z)|0≤z≤4−r2,0≤r≤1,0≤θ≤2π}E={(r,θ, z)|0≤z≤4−r2,0≤r≤1,0≤θ≤2π}


      Следовательно, интеграл для объема равен

      V(E)=∫θ=0θ=2π∫r=0r=1∫z=0z=4−r2rdzdrdθ=∫θ=0θ=2π∫r=0r=1[rz|z=0z=4−r2] drdθ=∫θ=0θ=2π∫r=0r=1(r4−r2)drdθ=∫02π(83−3)dθ=2π(83−3)кубических единиц.V(E)=∫θ=0θ=2π ∫r=0r=1∫z=0z=4−r2rdzdrdθ=∫θ=0θ=2π∫r=0r=1[rz|z=0z=4−r2]drdθ=∫θ=0θ=2π∫r=0r =1(r4−r2)drdθ=∫02π(83−3)dθ=2π(83−3)кубических единиц.

    2. Так как сфера равна x2+y2+z2=4,x2+y2+z2=4, то есть r2+z2=4,r2+z2=4, а цилиндр равен x2+y2=1,x2+y2=1 , то есть r2=1,r2=1, имеем 1+z2=4,1+z2=4, то есть z2=3.z2=3. Таким образом, у нас есть две области, так как сфера и цилиндр пересекаются в (1,3)(1,3) в rzrz-плоскости

      E1={(r,θ,z)|0≤r≤4−r2,3≤z≤2,0≤θ≤2π}E1={(r,θ,z)|0≤r≤4−r2 ,3≤z≤2,0≤θ≤2π}


      и

      E2={(r,θ,z)|0≤r≤1,0≤z≤3,0≤θ≤2π}.E2={ (r,θ,z)|0≤r≤1,0≤z≤3,0≤θ≤2π}.


      Следовательно, интеграл для объема равен

      V(E)=∫θ=0θ=2π∫z=3z=2∫r=0r=4−r2rdrdzdθ+∫θ=0θ=2π∫z=0z=3∫r =0r=1rdrdzdθ=3π+(163−33)π=2π(83−3)кубических единиц.V(E)=∫θ=0θ=2π∫z=3z=2∫r=0r=4−r2rdrdzdθ+∫ θ=0θ=2π∫z=0z=3∫r=0r=1rdrdzdθ=3π+(163−33)π=2π(83−3)кубических единиц.

    Контрольно-пропускной пункт 5.30

    Повторить предыдущий пример с порядком интегрирования dθdzdr.dθdzdr.

    Обзор сферических координат

    В трехмерном пространстве ℝ3ℝ3 в сферической системе координат мы задаем точку PP ее расстоянием ρρ от начала координат, полярным углом θθ от положительной оси xx (так же, как в цилиндрической системе координат) и угол φφ от положительной оси z z-оси и линии OPOP (рис. 5.55). Обратите внимание, что ρ≥0ρ≥0 и 0≤φ≤π.0≤φ≤π. (Обзор см. в разделе «Цилиндрические и сферические координаты».) Сферические координаты полезны для тройных интегралов по областям, симметричным относительно начала координат.

    Рисунок 5,55 Сферическая система координат находит точки с двумя углами и расстоянием от начала координат.

    Вспомните соотношения, связывающие прямоугольные координаты со сферическими координатами.

    От сферических координат к прямоугольным координатам:

    x=ρsinφcosθ,y=ρsinφsinθ,andz=ρcosφ.x=ρsinφcosθ,y=ρsinφsinθ,andz=ρcosφ.

    От прямоугольных координат к сферическим координатам:

    ρ2=x2+y2+z2,tanθ=yx,φ=arccos(zx2+y2+z2).ρ2=x2+y2+z2,tanθ=yx,φ=arccos(zx2+y2+z2).

    Другие отношения, которые важно знать для конверсий:

    •r=ρsinφ•θ=θЭти уравнения используются для преобразования сферических координат в цилиндрические•z=ρcosφ•r=ρsinφ•θ=θЭти уравнения используются для преобразования сферических координат в цилиндрические•z=ρcosφ

    и

    •ρ=r2+z2•θ=θЭти уравнения используются для преобразования цилиндрических координат в сферические координаты. •φ=arccos(zr2+z2)•ρ=r2+z2•θ=θЭти уравнения используются для преобразования цилиндрических координат в сферические координаты. • φ=arccos(zr2+z2)

    На следующем рисунке показаны несколько сплошных областей, которые удобно выразить в сферических координатах.

    Рисунок 5,56 Сферические координаты особенно удобны для работы с телами, ограниченными этими типами поверхностей. (Буква cc указывает на константу.)

    Интеграция в сферических координатах

    Теперь установим тройной интеграл в сферической системе координат, как мы это делали ранее в цилиндрической системе координат. Пусть функция f(ρ,θ,φ)f(ρ,θ,φ) непрерывна в ограниченном сферическом ящике, B={(ρ,θ,φ)|a≤ρ≤b,α≤θ≤β, γ≤φ≤ψ}.B={(ρ,θ,φ)|a≤ρ≤b,α≤θ≤β,γ≤φ≤ψ}. Затем мы делим каждый интервал на l,mandnl,mandn подразделений так, что Δρ=b−al,Δθ=β−αm, Δφ=ψ−γn.

    Теперь мы можем проиллюстрировать следующую теорему для тройных интегралов в сферических координатах с (ρijk*,θijk*,φijk*)(ρijk*,θijk*,φijk*) в качестве любой точки отсчета в сферическом подполе Bijk. Bijk. Для элемента объема подбокса ΔVΔV в сферических координатах имеем ΔV=(Δρ)(ρΔφ)(ρsinφΔθ), ΔV=(Δρ)(ρΔφ)(ρsinφΔθ), как показано на следующем рисунке.

    Рисунок 5,57 Элемент объема ящика в сферических координатах.

    Определение

    Тройной интеграл в сферических координатах является пределом тройной суммы Римана,

    liml,m,n→∞∑i=1l∑j=1m∑k=1nf(ρijk*,θijk*,φijk*)(ρijk*)2sinφΔρΔθΔφliml,m,n→∞∑i=1l∑j=1m ∑k=1nf(ρijk*,θijk*,φijk*)(ρijk*)2sinφΔρΔθΔφ

    при условии, что предел существует.

    Как и в случае с другими кратными интегралами, которые мы рассмотрели, все свойства работают аналогично для тройного интеграла в сферической системе координат, как и для повторных интегралов. Теорема Фубини принимает следующий вид.

    Теорема 5.13

    Теорема Фубини для сферических координат

    Если f(ρ,θ,φ)f(ρ,θ,φ) непрерывна на сферическом твердом ящике B=[a,b]×[α,β]×[γ,ψ],B=[a ,b]×[α,β]×[γ,ψ], тогда

    ∭Bf(ρ,θ,φ)ρ2sinφdρdφdθ=∫θ=αθ=β ∫φ=γφ=ψ∫ρ=aρ=bf(ρ ,θ,φ)ρ2sinφdρdφdθ. ∭Bf(ρ,θ,φ)ρ2sinφdρdφdθ=∫θ=αθ=β ∫φ=γφ=ψ∫ρ=aρ=bf(ρ,θ,φ)ρ2sinφdρdφdθ.

    (5.12)

    Этот повторный интеграл можно заменить другими повторными интегралами путем интегрирования по трем переменным в других порядках.

    Как указывалось ранее, сферические системы координат хорошо подходят для твердых тел, симметричных относительно точки, таких как сферы и конусы. Прежде чем рассматривать тройные интегралы в сферических координатах на общих сферических областях, рассмотрим несколько примеров.

    Пример 5,47

    Вычисление тройного интеграла в сферических координатах

    Вычисление повторного тройного интеграла 2∫p=0ρ=1ρ2sinφdρdφdθ.

    Решение

    Как и прежде, в этом случае переменные в повторном интеграле фактически независимы друг от друга, и, следовательно, мы можем интегрировать каждую часть и умножать: 2π)(1)(13)=2π3.∫02π∫0π/2∫01ρ2sinφdρdφdθ=∫02πdθ∫0π/2sinφdφ∫01ρ2dρ=(2π)(1)(13)=2π3.

    Концепция тройного интегрирования в сферических координатах может быть распространена на интегрирование по общему телу с использованием проекций на координатные плоскости. Обратите внимание, что dVdV и dAdA означают приращения объема и площади соответственно. Переменные VV и AA используются как переменные для интегрирования, чтобы выразить интегралы.

    Тройной интеграл непрерывной функции f(ρ,θ,φ)f(ρ,θ,φ) по сплошной области общего вида

    E={(ρ,θ,φ)|(ρ,θ)∈D,u1(ρ,θ)≤φ≤u2(ρ,θ)}E={(ρ,θ,φ)|(ρ, θ)∈D,u1(ρ,θ)≤φ≤u2(ρ,θ)}

    в ℝ3,ℝ3, где DD — проекция EE на плоскость ρθρθ, равна

    ∭Ef(ρ,θ,φ)dV=∬D[∫u1(ρ,θ)u2(ρ,θ)f (ρ,θ,φ)dφ]dA.∭Ef(ρ,θ,φ)dV=∬D[∫u1(ρ,θ)u2(ρ,θ)f(ρ,θ,φ)dφ]dA.

    В частности, если D={(ρ,θ)|g1(θ)≤ρ≤g2(θ),α≤θ≤β},D={(ρ,θ)|g1(θ)≤ρ≤g2 (θ),α≤θ≤β}, то имеем

    ∭Ef(ρ,θ,φ)dV=∫αβ∫g1(θ)g2(θ)∫u1(ρ,θ)u2(ρ,θ)f(ρ,θ,φ)ρ2sinφdφdρdθ.∭Ef(ρ ,θ,φ)dV=∫αβ∫g1(θ)g2(θ)∫u1(ρ,θ)u2(ρ,θ)f(ρ,θ,φ)ρ2sinφdφdρdθ.

    Аналогичные формулы встречаются и для проекций на другие координатные плоскости.

    Пример 5,48

    Составление тройного интеграла в сферических координатах

    Составление интеграла для объема области, ограниченной конусом z=3(x2+y2)z=3(x2+y2) и полусферой z=4−x2 −y2z=4−x2−y2 (см. рисунок ниже).

    Рисунок 5,58 Область, ограниченная снизу конусом и сверху полусферой.

    Решение

    Используя формулы преобразования прямоугольных координат в сферические координаты, имеем:

    Для конуса: z=3(x2+y2)z=3(x2+y2) или ρcosφ=3ρsinφρcosφ=3ρsinφ или tanφ=13tanφ=13 или φ=π6.φ=π6.

    Для сферы: z=4−x2−y2z=4−x2−y2 или z2+x2+y2=4z2+x2+y2=4 или ρ2=4ρ2=4 или ρ=2.ρ=2.

    Таким образом, тройной интеграл для объема V(E)=∫θ=0θ=2π∫ϕ=0φ=π/6∫ρ=0ρ=2ρ2sinφdρdφdθ.V(E)=∫θ=0θ=2π∫ϕ =0φ=π/6∫ρ=0ρ=2ρ2sinφdρdφdθ.

    Контрольно-пропускной пункт 5.31

    Составим тройной интеграл для объема твердой области, ограниченной сверху сферой ρ=2ρ=2 и ограниченной снизу конусом φ=π/3. φ=π/3.

    Пример 5,49

    Перестановка порядка интегрирования в сферических координатах

    Пусть EE — область, ограниченная снизу конусом z=x2+y2z=x2+y2 и сверху сферой z=x2+y2+z2z=x2+y2+z2 (рис. 5.59). Задайте тройной интеграл в сферических координатах и ​​найдите объем области, используя следующие порядки интегрирования:

    1. dρdϕdθ,dρdϕdθ,
    2. dφdρdθ.dφdρdθ.

      Рисунок 5,59 Область, ограниченная снизу конусом и сверху сферой.

    Решение
    1. Используйте формулы преобразования, чтобы записать уравнения сферы и конуса в сферических координатах.
      Для сферы:

      x2+y2+z2=zρ2=ρcosφρ=cosφ.x2+y2+z2=zρ2=ρcosφρ=cosφ.


      For the cone:

      z=x2+y2ρcosφ=ρ2sin2φcos2ϕ+ρ2sin2φsin2ϕρcosφ=ρ2sin2φ(cos2ϕ+sin2ϕ)ρcosφ=ρsinφcosφ=sinφφ=π/4.z=x2+y2ρcosφ=ρ2sin2φcos2ϕ+ρ2sin2φsin2ϕρcosφ=ρ2sin2φ(cos2ϕ+sin2ϕ)ρcosφ =ρsinφcosφ=sinφφ=π/4.


      Следовательно, интеграл для объема сплошной области EE становится равным

      V(E)=∫θ=0θ=2π∫φ=0φ=π/4∫ρ=0ρ=cosφρ2sinφdρdφdθ.V(E)=∫θ=0θ=2π∫φ=0φ=π/4∫ρ=0ρ =cosφρ2sinφdρdφdθ.

    2. Рассмотрим плоскость φρφρ. Обратите внимание, что диапазоны для φφ и ρρ (из части a.) составляют

      0≤φ≤π/40≤ρ≤cosφ.0≤φ≤π/40≤ρ≤cosφ.


      Кривая ρ=cosφρ=cosφ пересекает прямую φ=π/4φ=π/4 в точке (π/4,2/2).(π/4,2/2). Таким образом, для изменения порядка интегрирования нам нужно использовать две части:

      0≤ρ≤2/20≤φ≤π/4 и 2/2≤ρ≤10≤φ≤cos−1ρ.0≤ρ≤2/20 ≤φ≤π/4and2/2≤ρ≤10≤φ≤cos−1ρ.


      Следовательно, интеграл для объема сплошной области EE становится равным

      V(E)=∫θ=0θ=2π∫ρ=0ρ=2/2∫φ=0φ=π/4ρ2sinφdφdρdθ+∫θ=0θ=2π∫ρ=2/2ρ=1∫φ=0φ=cos− 1ρρ2sinφdφdρdθ.V(E)=∫θ=0θ=2π∫ρ=0ρ=2/2∫φ=0φ=π/4ρ2sinφdφdρdθ+∫θ=0θ=2π∫ρ=2/2ρ=1∫φ=0φ=cos −1ρρ2sinφdφdρdθ.


      В каждом случае интегрирование дает V(E)=π8.V(E)=π8.

    Прежде чем мы закончим этот раздел, мы приведем несколько примеров, которые могут проиллюстрировать преобразование прямоугольных координат в цилиндрические и из прямоугольных координат в сферические.

    Пример 5,50

    Преобразование прямоугольных координат в цилиндрические

    Преобразование следующего интеграла в цилиндрические координаты:

    ∫y=−1y=1∫x=0x=1−y2∫z=x2+y2z=x2+y2xyzdzdxdy.∫y= −1y=1∫x=0x=1−y2∫z=x2+y2z=x2+y2xyzdzdxdy.

    Решение

    Диапазоны переменных:

    −1≤y≤10≤x≤1−y2x2+y2≤z≤x2+y2.−1≤y≤10≤x≤1−y2x2+y2≤z≤x2+y2 .

    Первые два неравенства описывают правую половину окружности радиуса 1,1. Таким образом, диапазоны значений θθ и rr составляют

    −π2≤θ≤π2and0≤r≤1.−π2≤θ≤π2and0≤r≤1.

    Пределы zz равны r2≤z≤r,r2≤z≤r, следовательно,

    ∫y=−1y=1∫x=0x=1−y2∫z=x2+y2z=x2+y2xyzdzdxdy=∫θ =−π/2θ=π/2∫r=0r=1∫z=r2z=rr(rcosθ)(rsinθ)zdzdrdθ.∫y=−1y=1∫x=0x=1−y2∫z=x2+y2z =x2+y2xyzdzdxdy=∫θ=−π/2θ=π/2∫r=0r=1∫z=r2z=rr(rcosθ)(rsinθ)zdzdrdθ.

    Пример 5,51

    Преобразование прямоугольных координат в сферические координаты

    Преобразование следующего интеграла в сферические координаты:

    ∫y=0y=3∫x=0x=9−y2∫z=x2+y2z=18−x2−y2(x2+y2+z2)dzdxdy. ∫y=0y=3∫x=0x=9−y2∫z=x2+y2z=18−x2−y2( x2+y2+z2)dzdxdy.

    Решение

    Диапазоны переменных:

    0≤y≤30≤x≤9−y2x2+y2≤z≤18−x2−y2.0≤y≤30≤x≤9−y2x2+y2≤z≤18−x2 −y2.

    Первые два диапазона переменных описывают четверть диска в первом квадранте плоскости xyxy. Следовательно, диапазон для θθ составляет 0≤θ≤π2,0≤θ≤π2.

    Нижняя граница z=x2+y2z=x2+y2 — это верхняя половина конуса, а верхняя граница z=18−x2−y2z=18−x2−y2 — верхняя половина сферы. Следовательно, у нас есть 0≤ρ≤18,0≤ρ≤18, что равно 0≤ρ≤32,0≤ρ≤32.

    Для диапазонов φ,φ нам нужно найти, где пересекаются конус и сфера, поэтому решите уравнение

    r2+z2=18(x2+y2)2+z2=18z2+z2=182z2=18z2= 9z=3.r2+z2=18(x2+y2)2+z2=18z2+z2=182z2=18z2=9z=3.

    Это дает

    32cosφ=3cosφ=12φ=π4,32cosφ=3cosφ=12φ=π4.

    Собирая это вместе, мы получаем

    ∫y=0y=3∫x=0x=9−y2∫z=x2+y2z=18−x2−y2(x2+y2+z2)dzdxdy=∫φ=0φ= π/4∫θ=0θ=π/2∫ρ=0ρ=32ρ4sinφdρdθdφ.∫y=0y=3∫x=0x=9−y2∫z=x2+y2z=18−x2−y2(x2+y2+z2 )dzdxdy=∫φ=0φ=π/4∫θ=0θ=π/2∫ρ=0ρ=32ρ4sinφdρdθdφ.

    Контрольно-пропускной пункт 5.32

    Использование прямоугольных, цилиндрических и сферических координат для построения тройных интегралов для нахождения объема области внутри сферы x2+y2+z2=4×2+y2+z2=4, но вне цилиндра x2+y2=1.×2+ у2=1.

    Теперь, когда мы познакомились со сферической системой координат, давайте найдем объем некоторых известных геометрических фигур, таких как сферы и эллипсоиды.

    Пример 5,52

    Начало главы: поиск объема l’Hemisphèric

    Найдите объем сферического планетария в L’Hemisphèric в Валенсии, Испания, высотой в пять этажей и радиусом примерно 5050 футов, используя уравнение x2+y2+z2=r2.x2+y2+z2=r2 .

    Рисунок 5,60 (кредит: модификация работы Хавьера Яя Тура, Wikimedia Commons)

    Решение

    Рассчитаем объем шара в первом октанте, где x≥0,y≥0,x≥0,y≥0 и z≥0,z≥0, используя сферические координаты, а затем умножим результат на 88 для симметрии. Поскольку мы рассматриваем область DD как первый октант в интеграле, диапазоны значений переменных равны

    0≤φ≤π2,0≤ρ≤r,0≤θ≤π2.0≤φ≤π2,0≤ρ≤r,0≤θ≤π2.

    Следовательно,

    V=∭Ddxdydz=8∫θ=0θ=π/2∫ρ=0ρ=π∫φ=0φ=π/2ρ2sinθdφdρdθ=8∫φ=0φ=π/2dφ∫ρ=0ρ=rρ2dρ ∫θ=0θ=π/2sinθdθ=8(π2)(r33)(1)=43πr3.V=∭Ddxdydz=8∫θ=0θ=π/2∫ρ=0ρ=π∫φ=0φ=π/2ρ2sinθdφdρdθ =8∫φ=0φ=π/2dφ∫ρ=0ρ=rρ2dρ∫θ=0θ=π/2sinθdθ=8(π2)(r33)(1)=43πr3.

    Это точно соответствует тому, что мы знали. Таким образом, для сферы с радиусом примерно 5050 футов объем равен 43π(50)3≈523 600 футов3,43π(50)3≈523 600 футов3.

    Для следующего примера найдем объем эллипсоида.

    Пример 5,53

    Нахождение объема эллипсоида

    Нахождение объема эллипсоида x2a2+y2b2+z2c2=1.x2a2+y2b2+z2c2=1.

    Решение

    Мы снова используем симметрию и оцениваем объем эллипсоида, используя сферические координаты. Как и раньше, мы используем первый октант x≥0,y≥0,x≥0,y≥0 и z≥0z≥0, а затем умножаем результат на 8,8.

    В этом случае диапазоны значений переменных составляют

    0≤φ≤π2,0≤ρ≤π2,0≤ρ≤1, и 0≤θ≤π2,0≤φ≤π2,0≤ρ≤π2,0 ≤ρ≤1 и 0≤θ≤π2.

    Также нам нужно изменить прямоугольные координаты на сферические следующим образом:

    Тогда объем эллипсоида будет равен

    V=∭Ddxdydz=8∫θ=0θ=π/2∫ρ=0ρ=1∫φ=0φ=π/2abcρ2sinθdφdρdθ=8abc∫φ=0φ=π/2dφ∫ ρ=0ρ=1ρ2dρ∫θ=0θ=π/2sinθdθ=8abc(π2)(13)(1)=43πabc.V=∭Ddxdydz=8∫θ=0θ=π/2∫ρ=0ρ=1∫φ= 0φ=π/2abcρ2sinθdφdρdθ=8abc∫φ=0φ=π/2dφ∫ρ=0ρ=1ρ2dρ∫θ=0θ=π/2sinθdθ=8abc(π2)(13)(1)=43πabc.

    Пример 5,54

    Нахождение объема пространства внутри эллипсоида и вне сферы

    Найти объем пространства внутри эллипсоида x2752+y2802+z2902=1×2752+y2802+z2902=1 и вне сферы x2+y2+z2=502 .x2+y2+z2=502.

    Решение

    Эта проблема напрямую связана со структурой полушария. Объем пространства внутри эллипсоида и вне сферы может быть полезен для определения затрат на нагрев или охлаждение этого пространства. Мы можем использовать предыдущие два примера для объема сферы и эллипсоида, а затем вычесть.

    Сначала мы найдем объем эллипсоида, используя a=75ft,a=75ft,b=80ft,b=80ft и c=90ftc=90ft в результате примера 5.53. Следовательно, объем эллипсоида равен

    .

    Из примера 5.52 объем сферы равен

    Vсфера≈523 600 футов3.Vсфера≈523 600 футов3.

    Следовательно, объем пространства внутри эллипсоида x2752+y2802+z2902=1×2752+y2802+z2902=1 и вне сферы x2+y2+z2=502×2+y2+z2=502 примерно равен

    VПолушарие=Вэллипсоид-Vсфера=1 738 400 футов3.

    Студенческий проект

    Воздушные шары

    Полеты на воздушном шаре — это расслабляющее и мирное времяпрепровождение, которое нравится многим людям. Многие сборы воздухоплавателей проходят по всему миру, например, Международная фиеста воздушных шаров в Альбукерке. Мероприятие в Альбукерке является крупнейшим фестивалем воздушных шаров в мире, в котором ежегодно участвует более 500 500 воздушных шаров.

    Рисунок 5,61 Воздушные шары взлетают на Международном празднике воздушных шаров в Альбукерке в 2001–2001 годах. (Фото: Дэвид Эррера, Flickr)

    Как следует из названия, воздушные шары используют горячий воздух для создания подъемной силы. (Горячий воздух менее плотный, чем более холодный воздух, поэтому воздушный шар парит, пока горячий воздух остается горячим.) Тепло вырабатывается пропановой горелкой, подвешенной под отверстием корзины. Как только воздушный шар взлетает, пилот контролирует высоту воздушного шара либо с помощью горелки для нагрева воздуха и подъема, либо с помощью вентиляционного отверстия в верхней части воздушного шара для выпуска нагретого воздуха и спуска. Однако у пилота очень мало контроля над тем, куда полетит воздушный шар — воздушные шары находятся во власти ветров. Неуверенность в том, где мы закончим, — одна из причин, по которой воздухоплавателей привлекает этот вид спорта.

    В этом проекте мы используем тройные интегралы, чтобы узнать больше о воздушных шарах. Мы моделируем воздушный шар из двух частей. Верхняя часть воздушного шара моделируется полусферой радиусом 2828 футов. Нижняя часть воздушного шара смоделирована в виде усеченного конуса (представьте себе конус мороженого с отрезанным заостренным концом). Радиус большого конца усеченного конуса составляет 2828 футов, а радиус маленького конца усеченного конуса — 66 футов. График нашей модели воздушного шара и диаграмма поперечного сечения, показывающая размеры, показаны на следующем рисунке.

    Рисунок 5,62 (a) Используйте полусферу для моделирования верхней части воздушного шара и усеченный конус для моделирования нижней части воздушного шара. (b) Поперечное сечение воздушного шара с указанием его размеров.

    Сначала мы хотим найти объем воздушного шара. Если мы посмотрим на верхнюю и нижнюю часть шара по отдельности, то увидим, что они представляют собой геометрические тела с известными формулами объема. Тем не менее, все же стоит установить и вычислить интегралы, которые нам потребуются для нахождения объема. Если мы рассчитаем объем с помощью интегрирования, мы можем использовать известные формулы объема для проверки наших ответов. Это поможет обеспечить правильную настройку интегралов для более поздних, более сложных этапов проекта.

    1. Найдите объем воздушного шара двумя способами.
      1. Используйте тройные интегралы для вычисления объема. Рассмотрим каждую часть воздушного шара отдельно. (Рассмотрите возможность использования сферических координат для верхней части и цилиндрических координат для нижней части.)
      2. Проверьте ответ, используя формулы для объема сферы V=43πr3,V=43πr3 и объема конуса V=13πr2h.V=13πr2h.
      На самом деле вычисление температуры в точке внутри воздушного шара — чрезвычайно сложная задача. На самом деле целая ветвь физики (термодинамика) посвящена изучению теплоты и температуры. Однако для целей этого проекта мы собираемся сделать некоторые упрощающие предположения о том, как температура меняется от точки к точке внутри воздушного шара. Предположим, что непосредственно перед взлетом температура (в градусах Фаренгейта) воздуха внутри шара изменяется в соответствии с функцией

      T0(r,θ,z)=z−r10+210.T0(r,θ,z)=z−r10+210.

    2. Какова средняя температура воздуха в воздушном шаре непосредственно перед стартом? (Опять же, смотрите на каждую часть шара отдельно, и не забудьте преобразовать функцию в сферические координаты, глядя на верхнюю часть шара.)
      Теперь пилот включает горелку на 1010 секунд. Это действие влияет на температуру в колонне шириной 1212 футов и высотой 2020 футов, прямо над горелкой. Поперечное сечение воздушного шара, изображающего эту колонну, показано на следующем рисунке.

      Рисунок 5,63 Активация горелки нагревает воздух в колонне высотой 2020 футов и шириной 1212 футов прямо над горелкой.


      Предположим, что после включения пилотом горелки в течение 1010 секунд температура воздуха в описанной выше колонне возрастает по формуле

      H(r,θ,z)=−2z−48. H(r, θ,z)=−2z−48.


      Тогда температура воздуха в колонне определяется выражением

      T1(r,θ,z)=z−r10+210+(−2z−48),T1(r,θ,z)=z−r10+ 210+(−2z−48),


      , в то время как температура в остальной части баллона по-прежнему определяется как

      T0(r,θ,z)=z−r10+210.T0(r,θ,z)=z−r10+210.

    3. Найдите среднюю температуру воздуха в воздушном шаре после того, как пилот включил горелку на 1010 секунд.

    Раздел 5.5 Упражнения

    В следующих упражнениях оцените тройные интегралы ∭Ef(x,y,z)dV∭Ef(x,y,z)dV по твердому телу B.B.

    241.

    f(x,y,z)=z,f(x,y,z)=z,B={(x,y,z)|x2+y2≤9,x≥0,y≥0,0≤z≤1}B={(x,y,z)|x2+y2≤9,x≥0,y≥0,0≤z≤1}

    242.

    f(x,y,z)=xz2,f(x,y,z)=xz2,B={(x,y,z)|x2+y2≤16,x≥0,y≤0,− 1≤z≤1}B={(x,y,z)|x2+y2≤16,x≥0,y≤0,−1≤z≤1}

    243.

    f(x,y,z)=xy,f(x,y,z)=xy,B={(x,y,z)|x2+y2≤1,x≥0,x≥y,− 1≤z≤1}B={(x,y,z)|x2+y2≤1,x≥0,x≥y,−1≤z≤1}

    244.

    f(x,y,z)=x2+y2,f(x,y,z)=x2+y2,B={(x,y,z)|x2+y2≤4,x≥0,x ≤y,0≤z≤3}B={(x,y,z)|x2+y2≤4,x≥0,x≤y,0≤z≤3}

    245.

    f(x,y,z)=ex2+y2,f(x,y,z)=ex2+y2,B={(x,y,z)|1≤x2+y2≤4,y≤0 ,x≤y3,2≤z≤3}B={(x,y,z)|1≤x2+y2≤4,y≤0,x≤y3,2≤z≤3}

    246.

    f(x,y,z)=x2+y2,f(x,y,z)=x2+y2,B={(x,y,z)|1≤x2+y2≤9,y≤0 ,0≤z≤1}B={(x,y,z)|1≤x2+y2≤9,y≤0,0≤z≤1}

    247.

    1. Пусть BB — цилиндрическая оболочка с внутренним радиусом a,a, внешним радиусом b,b и высотой c,c, где 00.c>0. Предположим, что функция FF, заданная на BB, может быть выражена в цилиндрических координатах как F(x,y,z)=f(r)+h(z),F(x,y,z)=f(r)+h( z), где ff и hh — дифференцируемые функции. Если ∫abf˜(r)dr=0∫abf˜(r)dr=0 и h˜(0)=0,h˜(0)=0, где f˜f˜ и h˜h˜ — первообразные функции ff и h,h соответственно показывают, что

      ∭BF(x,y,z)dV=2πc(bf˜(b)−af˜(a))+π(b2−a2)h˜(c).∭BF(x,y,z)dV= 2πc(bf˜(b)−af˜(a))+π(b2−a2)h˜(c).

    2. Используйте предыдущий результат, чтобы показать, что ∭B(z+sinx2+y2)dxdydz=6π2(π−2),∭B(z+sinx2+y2)dxdydz=6π2(π−2), где BB — цилиндрическая оболочка. с внутренним радиусом π, π, внешним радиусом 2π, 2π и высотой 2,2.

    248.

    1. Пусть BB — цилиндрическая оболочка с внутренним радиусом a,a, внешним радиусом b,b и высотой c,c, где 00.c>0. Предположим, что функция FF, заданная на BB, может быть выражена в цилиндрических координатах как F(x,y,z)=f(r)g(θ)h(z),F(x,y,z)=f(r) g(θ)h(z), где f,g,andhf,g,h — дифференцируемые функции. Если ∫abf˜(r)dr=0,∫abf˜(r)dr=0, где f˜f˜ — первообразная f,f, покажите, что

      ∭BF(x,y,z)dV=[bf˜(b)−af˜(a)][g˜(2π)−g˜(0)][h˜(c)−h˜(0) ],∭BF(x,y,z)dV=[bf˜(b)−af˜(a)][g˜(2π)−g˜(0)][h˜(c)−h˜(0 )],


      , где g˜g˜ и h˜h˜ — первообразные gg и h,h соответственно.
    2. Используйте предыдущий результат, чтобы показать, что ∭Bzsinx2+y2dxdydz=−12π2,∭Bzsinx2+y2dxdydz=−12π2, где BB — цилиндрическая оболочка с внутренним радиусом π,π, внешним радиусом 2π,2π и высотой 2,2.

    В следующих упражнениях границы сплошного EE задаются в цилиндрических координатах.

    1. Задайте область в цилиндрических координатах.
    2. Преобразуйте интеграл ∭Ef(x,y,z)dV∭Ef(x,y,z)dV в цилиндрические координаты.

    249.

    EE ограничен правым круговым цилиндром r=4sinθ,r=4sinθ, плоскостью rθrθ и сферой r2+z2=16.r2+z2=16.

    250.

    EE находится вне правого кругового цилиндра r=cosθ,r=cosθ, над плоскостью x–yx–y и внутри сферы r2+z2=9.r2+z2=9.

    251.

    EE расположен в первом октанте и ограничен круговым параболоидом z=9−3r2,z=9−3r2, цилиндр r=3,r=3 и плоскость r(cosθ+sinθ)=20−z.r(cosθ+sinθ)=20−z.

    252.

    EE расположен в первом октанте вне кругового параболоида z=10−2r2z=10−2r2 и внутри цилиндра r=5r=5 и ограничен также плоскостями z=20z=20 и θ=π4. θ= №4.

    В следующих упражнениях даны функция ff и область EE.

    1. Выразите область EE и функцию ff в цилиндрических координатах.
    2. Преобразуйте интеграл ∭Bf(x,y,z)dV∭Bf(x,y,z)dV в цилиндрические координаты и вычислите его.

    253.

    f(x,y,z)=1x+3,f(x,y,z)=1x+3,E={(x,y,z)|0≤x2+y2≤9,x≥0 ,y≥0,0≤z≤x+3}E={(x,y,z)|0≤x2+y2≤9,x≥0,y≥0,0≤z≤x+3}

    254.

    f(x,y,z)=x2+y2,f(x,y,z)=x2+y2,E={(x,y,z)|0≤x2+y2≤4,y≥0 ,0≤z≤3−x}E={(x,y,z)|0≤x2+y2≤4,y≥0,0≤z≤3−x}

    255.

    f(x,y,z)=x,f(x,y,z)=x,E={(x,y,z)|1≤y2+z2≤9,0≤x≤1−y2 −z2}E={(x,y,z)|1≤y2+z2≤9,0≤x≤1−y2−z2}

    256.

    f(x,y,z)=y,f(x,y,z)=y,E={(x,y,z)|1≤x2+z2≤9,0≤y≤1−x2 −z2}E={(x,y,z)|1≤x2+z2≤9,0≤y≤1−x2−z2}

    В следующих упражнениях найдите объем твердого EE, границы которого заданы в прямоугольных координатах.

    257.

    EE находится над плоскостью xyxy, внутри цилиндра x2+y2=1,x2+y2=1 и ниже плоскости z=1.z=1.

    258.

    EE находится ниже плоскости z=1z=1 и внутри параболоида z=x2+y2.z=x2+y2.

    259.

    EE ограничен круговым конусом z=x2+y2z=x2+y2 и z=1.z=1.

    260.

    EE расположен выше плоскости xyxy, ниже z=1,z=1, вне однополостного гиперболоида x2+y2−z2=1,x2+y2−z2=1 и внутри цилиндра x2+y2= 2.х2+у2=2.

    261.

    EE расположен внутри цилиндра x2+y2=1×2+y2=1 и между круговыми параболоидами z=1−x2−y2z=1−x2−y2 и z=x2+y2.z=x2+y2.

    262.

    EE находится внутри сферы x2+y2+z2=1,x2+y2+z2=1, над плоскостью xyxy и внутри кругового конуса z=x2+y2.z=x2+y2.

    263.

    EE расположен внутри кругового конуса x2+y2=(z−1)2×2+y2=(z−1)2 и между плоскостями z=0z=0 и z=2. z=2.

    264.

    EE расположен вне кругового конуса z=1−x2+y2,z=1−x2+y2, выше плоскости xyxy, ниже кругового параболоида и между плоскостями z=0 и z=2.z=0andz= 2.

    265.

    [T] Используйте систему компьютерной алгебры (CAS) для построения графика твердого тела, объем которого определяется повторным интегралом в цилиндрических координатах ∫−π/2π/2∫01∫r2rrdzdrdθ.∫−π/2π/2∫ 01∫r2rrdzdrdθ. Найдите объем VV тела. Округлите ответ до четырех знаков после запятой.

    266.

    [T] Используйте CAS для построения графика твердого тела, объем которого определяется повторным интегралом в цилиндрических координатах ∫0π/2∫01∫r4rrdzdrdθ.∫0π/2∫01∫r4rrdzdrdθ. Найдите объем VV твердого тела Округлите ответ до четырех знаков после запятой.

    267.

    Преобразовать интеграл ∫01∫−1−y21−y2∫x2+y2x2+y2xzdzdxdy∫01∫−1−y21−y2∫x2+y2x2+y2xzdzdxdy в интеграл в цилиндрических координатах.

    268.

    Преобразовать интеграл ∫02∫0y∫01(xy+z)dzdxdy∫02∫0y∫01(xy+z)dzdxdy в интеграл в цилиндрических координатах.

    В следующих упражнениях вычислите тройной интеграл ∭Bf(x,y,z)dV∭Bf(x,y,z)dV по твердому B.B.

    269.

    f(x,y,z)=1,f(x,y,z)=1,B={(x,y,z)|x2+y2+z2≤90,z≥0}B={ (x,y,z)|x2+y2+z2≤90,z≥0}

    270.

    f(x,y,z)=1−x2+y2+z2,f(x,y,z)=1−x2+y2+z2,B={(x,y,z)|x2+y2 +z2≤9,y≥0,z≥0}B={(x,y,z)|x2+y2+z2≤9,y≥0,z≥0}

    271.

    f(x,y,z)=x2+y2,f(x,y,z)=x2+y2,BB ограничен сверху полусферой x2+y2+z2=9×2+y2+z2=9 с z≥0z≥0 и ниже конусом 2z2=x2+y2.2z2=x2+y2.

    272.

    f(x,y,z)=z,f(x,y,z)=z,BB ограничен сверху полусферой x2+y2+z2=16×2+y2+z2=16 с z≥0z ≥0 и ниже конусом 2z2=x2+y2.2z2=x2+y2.

    273.

    Показать, что если F(ρ,θ,φ)=f(ρ)g(θ)h(φ)F(ρ,θ,φ)=f(ρ)g(θ)h(φ) является непрерывным функция на сферическом ящике B={(ρ,θ,φ)|a≤ρ≤b,α≤θ≤β,γ≤φ≤ψ},B={(ρ,θ,φ)|a≤ρ≤ b,α≤θ≤β,γ≤φ≤ψ}, тогда

    ∭BFdV=(∫abρ2f(ρ)dr)(∫αβg(θ)dθ)(∫γψh(φ)sinφdφ). ∭BFdV=( ∫abρ2f(ρ)dr)(∫αβg(θ)dθ)(∫γψh(φ)sinφdφ).

    274.

    1. Говорят, что функция FF имеет сферическую симметрию, если она зависит только от расстояния до начала координат, то есть ее можно выразить в сферических координатах как F(x,y,z)=f(ρ),F( x,y,z)=f(ρ), где ρ=x2+y2+z2.ρ=x2+y2+z2. Покажите, что

      ∭BF(x,y,z)dV=2π∫abρ2f(ρ)dρ, ∭BF(x,y,z)dV=2π∫abρ2f(ρ)dρ,


      , где BB — область между верхние концентрические полусферы радиусов aa и bb с центром в начале координат, с 0
    2. Используйте предыдущий результат, чтобы показать, что ∭B(x2+y2+z2)x2+y2+z2dV=21π,∭B(x2+y2+z2)x2+y2+z2dV=21π, где

      B={(x, y,z)|1≤x2+y2+z2≤2,z≥0}.B={(x,y,z)|1≤x2+y2+z2≤2,z≥0}.

    275.

    1. Пусть BB — область между верхними концентрическими полусферами радиусов a и b с центром в начале координат и расположенная в первом октанте, где 0 0 F функцию, определенную на B , форма которой в сферических координатах (ρ, θ, φ)(ρ, θ, φ) равна F(x,y,z)=f(ρ)cosφ.F(x, y,z)=f(ρ)cosφ. Покажите, что если g(a)=g(b)=0g(a)=g(b)=0 и ∫abh(ρ)dρ=0,∫abh(ρ)dρ=0, то

      ∭BF(x,y,z)dV=π24[ah(a)−bh(b)],∭BF(x,y,z)dV=π24[ah(a)−bh(b)],


      , где gg — производная от ff, а hh — производная от g.g.
    2. Используйте предыдущий результат, чтобы показать, что ∭Bzcosx2+y2+z2x2+y2+z2dV=3π22,∭Bzcosx2+y2+z2x2+y2+z2dV=3π22, где BB — область между верхними концентрическими полусферами радиусов ππ и 2π2π с центром в начале и расположен в первом октант.

    В следующих упражнениях даны функция ff и область EE.

    1. Выразите область EE и функцию ff в сферических координатах.
    2. Преобразуйте интеграл ∭Bf(x,y,z)dV∭Bf(x,y,z)dV в сферические координаты и вычислите его.

    276.

    f(x,y,z)=z;f(x,y,z)=z;E={(x,y,z)|0≤x2+y2+z2≤1,z≥0}E ={(x,y,z)|0≤x2+y2+z2≤1,z≥0}

    277.

    f(x,y,z)=x+y;f(x,y,z)=x+y;E={(x,y,z)|1≤x2+y2+z2≤2,z ≥0,y≥0}E={(x,y,z)|1≤x2+y2+z2≤2,z≥0,y≥0}

    278.

    f(x,y,z)=2xy;f(x,y,z)=2xy;E={(x,y,z)|x2+y2≤z≤1−x2−y2,x≥0 ,y≥0}E={(x,y,z)|x2+y2≤z≤1−x2−y2,x≥0,y≥0}

    279.

    f(x,y,z)=z;f(x,y,z)=z;E={(x,y,z)|x2+y2+z2−2z≤0,x2+y2≤z }E={(x,y,z)|x2+y2+z2−2z≤0,x2+y2≤z}

    В следующих упражнениях найдите объем твердого EE, границы которого заданы в прямоугольных координатах.

    280.

    E={(x,y,z)|x2+y2≤z≤16−x2−y2,x≥0,y≥0}E={(x,y,z)|x2+y2≤z≤16− х2-у2,х≥0,у≥0}

    281.

    E={(x,y,z)|x2+y2+z2−2z≤0,x2+y2≤z}E={(x,y,z)|x2+y2+z2−2z≤0,x2+ у2≤г}

    282.

    Используйте сферические координаты, чтобы найти объем твердого тела, расположенного внутри сферы ρ=1ρ=1 и вне сферы ρ=cosφ,ρ=cosφ, где φ∈[0,π2].φ∈[0,π2].

    283.

    Используя сферические координаты, найдите объем шара ρ≤3ρ≤3, расположенного между конусами φ=π4и φ=π3.φ=π4иφ=π3.

    284.

    Преобразовать интеграл y216−x2−y2(x2+y2+z2)dzdxdy в интеграл в сферических координатах.

    285.

    Преобразовать интеграл y2+z2)2dzdydx в интеграл в сферических координатах.

    286.

    Преобразовать интеграл ∫−22∫−4−x24−x2∫x2+y216−x2−y2dzdydx∫−22∫−4−x24−x2∫x2+y216−x2−y2dzdydx в интеграл в сферических координатах и ​​вычислить его .

    287.

    [T] Используйте CAS для построения графика твердого тела, объем которого определяется повторным интегралом в сферических координатах . Найдите объем VV тела. Округлите ответ до трех знаков после запятой.

    288.

    [T] Используйте CAS для построения графика твердого тела, объем которого определяется повторным интегралом в сферических координатах как ∫02π∫π/43π/4∫01ρ2sinφdρdφdθ. ∫02π∫π/43π/4∫01ρ2sinφdρdφdθ. Найдите объем VV тела. Округлите ответ до трех знаков после запятой.

    289.

    [T] Используйте CAS для вычисления интеграла ∭E(x2+y2)dV∭E(x2+y2)dV, где EE лежит над параболоидом z=x2+y2z=x2+y2 и ниже плоскости z =3y.z=3y.

    290.

    [Т]

    1. Вычислите интеграл ∭Eex2+y2+z2dV,∭Eex2+y2+z2dV, где EE ограничен сферами 4×2+4y2+4z2=14×2+4y2+4z2=1 и x2+y2+z2=1.×2 +y2+z2=1.
    2. Используйте CAS, чтобы найти аппроксимацию предыдущего интеграла. Округлите ответ до двух знаков после запятой.

    291.

    Выразите объем твердого тела внутри сферы x2+y2+z2=16×2+y2+z2=16 и вне цилиндра x2+y2=4×2+y2=4 тройными интегралами в цилиндрических координатах и ​​сферических координатах.

    292.

    Выразите объем тела внутри сферы x2+y2+z2=16×2+y2+z2=16 и вне цилиндра x2+y2=4×2+y2=4, расположенного в первом октанте, тройными интегралами в цилиндрических координатах и сферические координаты.

    293.

    Мощность, излучаемая антенной, имеет плотность мощности на единицу объема, заданную в сферических координатах как

    единицы в ваттах. Полная мощность в пределах сферы BB радиуса rr метров определяется как P=∭Bp(ρ,θ,φ)dV.P=∭Bp(ρ,θ,φ)dV. Найдите полную мощность ПП в сфере радиусом 20 метров.

    294.

    Используйте предыдущее упражнение, чтобы найти полную мощность в сфере BB радиусом 5 метров, когда плотность мощности на единицу объема определяется выражением p(ρ,θ,φ)=30ρ2cos2θsin4φ.p(ρ,θ,φ)=30ρ2cos2θsin4φ .

    295.

    Облако заряда, содержащееся в сфере BB радиусом r сантиметров с центром в начале координат, имеет плотность заряда, определяемую выражением q(x,y,z)=kx2+y2+z2µCcm3,q(x,y,z)= kx2+y2+z2µCcm3, где k>0.k>0. Полный заряд, содержащийся в BB, равен Q=∭Bq(x,y,z)dV.Q=∭Bq(x,y,z)dV. Найдите общий заряд Q. Q.

    296.

    Используйте предыдущее упражнение, чтобы найти полное облако заряда, содержащееся в единичной сфере, если плотность заряда q(x,y,z)=20×2+y2+z2µCcm3.q(x,y,z)=20×2+y2+ z2мксм3.

    Калькулятор тройных интегралов / Решатель с бесплатными шагами

    Онлайн-калькулятор тройных интегралов поможет вам определить значения тройных интегралов заданной функции. Калькулятор цилиндрического интеграла оценивает тройной интеграл несколькими методами и отображает пошаговые вычисления. В этой статье вы узнаете, как вычислять тройные интегралы и многое другое.

    Что такое тройной интеграл?

    В математике тройной интеграл аналогичен одинарному или двойному интегралу. Обычно тройное интегрирование используется для интегрирования по трехмерному пространству. Тройной интеграл используется для определения объема подобно двойному интегралу. Но она же определяет и массу, когда объем любого тела имеет переменную плотность. Функция может быть выражена как:

    $$ F (x, y, z) $$

    Интеграл вычисляется в зависимости от порядка записи и того, как задана конкретная запись.

    Например, порядок dxdydz, затем интегрирование по x, затем y, затем z. Повторные интегралы вычисляются от самого внутреннего к самому внешнему.

    Как вычислить тройной интеграл?

    В тройном интеграле вам необходимо оценить интегрирование трех переменных по отношению к трем различным переменным. Вы должны изучить следующий модуль, чтобы понять, как вычислять тройной интеграл. Выполните следующие шаги и определите функции с помощью калькулятора тройного интеграла или вручную.

    • Возьмите функцию с тремя разными переменными, чтобы вычислить тройной интеграл.
    • Сначала выполните интегрирование с одной переменной, чтобы исключить определенную переменную.
    • Теперь подставьте полученные значения в выражение типа верхний и нижний предел.
    • При интегрировании одной переменной необходимо считать остальные переменные постоянными.
    • После исключения одной переменной вы должны повторить процесс, чтобы исключить другие переменные, чтобы получить ответ в константе. 92 yz (8x + 3yz (2z + 1)) / 24 + константа $$

      Интегрирование в цилиндрических координатах:

      Тройные интегралы обычно вычисляются с использованием цилиндрических координат, а не прямоугольных. Некоторые уравнения в прямоугольных координатах вместе с родственными уравнениями в цилиндрических координатах приведены в табл. Уравнения становятся проще, когда калькулятор цилиндрических интегралов приступает к решению задач с использованием тройных интегралов.

      92) \)

      Однако онлайн-калькулятор производных помогает определить производную функции по заданной переменной.

      Как работает калькулятор тройного интеграла?

      Онлайн-калькулятор тройных интегралов может найти предел суммы произведения функции, выполнив следующие действия:

      Ввод:
      • Сначала введите функцию относительно переменных x, y и z .
      • Если известны верхний и нижний пределы переменных, то выберите определенный и подставьте верхний и нижний пределы.
      • Кроме того, если вы понятия не имеете о пределах переменных, то выберите неопределенное.
      • Нажмите кнопку «Вычислить тройной интеграл».

      Вывод:
      • Цилиндрический интегральный калькулятор отображает неопределенный и определенный интеграл с пошаговым вычислением.

      Часто задаваемые вопросы:

      Почему можно использовать тройной интеграл?

      Тройной интеграл в основном используется для определения массы и объема, как и двойной интеграл.

      Для чего используется объемный интеграл?

      В исчислении объемный интеграл относится к интегралу по трехмерной области. Это частный случай кратных интегралов. Для расчета плотностей потоков в физике чаще всего используется объемный интеграл.

      Как найти объем тройного интеграла? 92sin θ dpdφdθ \). По симметрии можно оценить объем эллипсоида, лежащего в первом октанте, и умножить результат на 8.

      Вывод:

      Используйте этот онлайн-калькулятор тройного интеграла для определения тройного интеграла от введенных функций. Тройной интеграл в основном используется для определения объема и массы функций с тремя различными переменными, которые необходимо интегрировать на заданном интервале.

      Ссылка:

      Из источника Википедии: Кратный интеграл, Интегрируемость по Риману, Методы интегрирования, Интегрирование постоянных функций, Использование симметрии, Нормальные области на R2.
      Из источника Libre Text: Тройные интегралы в цилиндрических координатах, Интегрирование в цилиндрических координатах, Теорема Фубини в цилиндрических координатах.
      Из источника Lumen Learning: Двойные интегралы по прямоугольникам, Двойные интегралы по прямоугольникам, Повторные интегралы, Двойные интегралы по обычным областям.

      Тройные интегралы в цилиндрических координатах

      Тройные интегралы в цилиндрических координатах

      Закон обратных квадратов

      Предположим, две точечные массы с массами м и м соответственно расположенных на расстоянии r друг от друга. Закон обратных квадратов сэра Исаака Ньютона утверждает, что величина | Ф | силы гравитации между двумя точечными массами

      | Ф | = Г мм р 2
      (4)
      где G — универсальная гравитационная постоянная. Однако, как Ньютон осознали и какое-то время боролись с ними, объекты в реальном мире не точечных масс, и вместо этого может потребоваться закон (4). модифицированный.

      В частности, предположим, что одно из тел не »массовая точка», но вместо этого представляет собой сферу радиусом R с однородной массой плотность м. Предположим, что r > R и что сфера с центром в (0,0, r ) . Если другое тело является точечной массой »спутник» массой м , расположенный в начале координат, то гравитационное сила направлена ​​вдоль оси z .

      Предположим теперь, что небольшой «кусок» сферы находится в точке ( r,f,q) (в сферических координатах), и пусть что он имеет небольшую массу дМ . Тогда расстояние между маленьким кусочком и происхождение р.
      так что по (4) «малая» звездная величина d | Ф | силы гравитации между маленьким «кусочком» и спутник
      д | Ф | = Гм    дМ р 2
      (5)
      Однако из симметрии сферы следует, что нам нужны только гравитационные потяните в вертикальном направлении. Сумма д | Ф | в вертикальном направлении определяется выражением cos(f) d | Ф | (см. выше).

      Таким образом, полная гравитационная сила в вертикальном направлении равна

      | Ф | =


      С  

      cos( f) d | Ф | =


      С  

      Gm   cos(f) р 2
      дМ
      где S — сфера, соответствующая «планете». Если dV обозначает объем небольшого «кусочка» шара, тогда дм = м dV , что приводит к
      | Ф | = — Гм м


      С  

      потому что (f) р 2
      дВ
      (6)

      В декартовых координатах сфера S задается как

      х 2 + y 2 +( z r ) 2 = R 2         or         x 2 + y 2 + z 2 -2 рз + р 2 = р 2
      В сферических координатах это становится
      .
      р 2 -2 р rcos(f) + r 2 R 2 = 0
      что по квадратичной формуле приводит к
      r cos(f)   
      R 2 r 2 (1-cos 2 (f))
      r cos(f)   
      R 2 r 2 син 2 (f)
      Таким образом, сфера находится между
      r 1 = r cosf —  
      R 2 r 2 sin 2 (f)
      и     r 2 = r cosf +  
      R 2 r 2 син 2 (f)
      Отметим также, что f находится в диапазоне от 0 до sin -1 ( R / r ), а q находится в диапазоне от [ 0,2p] .

      Вычисление (6) в сферических координатах приводит к

      55
      944139
      Гм м
      2п



      sin -1 ( R / r )



      р 2

      р 1  

      потому что (f) р 2
      r 2 sin(f) d r d f d q
      Гм м
      2п



      sin -1 ( R / r )



      р 2

      р 1  

      cos( f) sin( f) d r d f d q
      Гм м
      2п



      sin -1 ( р / р )


      ( r 2 -r 1 ) cos( f) sin( f) d r d f d q
      Со времени R 2 -R 1 = 2 ( R 2 R 2 SIN 2 (F)) 1/2 , это в свою очередь приводит к
      5555555 1/2 , это в очереди приводит к
      55 1/2 , это срока.
      | Ф | = -2 Гм м
      2п



      sin -1 ( R / r )


      ( R 2 r 2 sin 2 ( f) ) 1/2 sin( f) cos( f) d f d q
      Если положить u (f) = R 2 р 2 син 2 (ж) , тогда пределы интегрирования становятся
      u ( 0) = R 2         и         u
      грех -1
      Р р


      = р 2 р 2
      р 2 р 2

      = 0
      Кроме того, du = -2 r 2 2sin( f) cos( f) d f, так что
      Гм м р 2

      2п



      sin -1 ( R / r )


      ( R 2 r 2 sin 2 ( f)) 1/2   ( -2 r 2 ( f) cos s0003 d f d q
      Гм м р 2

      2п



      0

      Ч 2  

      u 1/2 du    д д
      Гм м р 2

      2п


      u 3/2 3/2

      0

      Ч 2  

      д д
      Гм м р 2

      2п


      2 Р 3 3
      д д
      Гм р 2
      м 4п R 3 3
      Однако объем сферы равен V = 4p R 3 /3, так что масса сфера M = m V = m 4p R 3 /3.

Добавить комментарий

Ваш адрес email не будет опубликован. Обязательные поля помечены *